SlideShare una empresa de Scribd logo
1 de 134
NEFROLOGIA
ENARM
ANION GAP
• El Anión GAP es una ECUACIÓN que sirve PARA CALCULAR ANIONES ORGÁNICOS que están presentes a
una CONCENTRACIÓN MUY PEQUEÑA como para ser medidas en un ionograma...
Partiendo desde el concepto de electro neutralidad, pareciera que los cationes "superan" a los aniones,
pero justamente la "brecha" entre ellos esta representados por estos Aniones...
La utilidad fisiológica/clínica del Anión Gap, es para orientarte acerca de la POSIBLE ETIOLOGÍA ( causa)
DE LA ACIDOSIS METABOLICA ( y SOLO SIRVE PARA ESTO) que el paciente esta cursando...
Las AC METAB. pueden ser DE 2 "TIPOS"... por FALTA DE BASE ( el tipo tiene una DIARREA tremenda, esta
largando hasta la 1º papilla... ), o por EXCESO DE ACIDO ( esta TOMANDO ASPIRINA como si fueran tic
tacs... o CETOACIDOSIS etc.) ( esos son algunos ejemplos... )...
Si calculas el AG... SIENDO AG= [NA+]P - ([CL-]P+[HCO3-]P)
Los valores NORMALES: AG= 12 +- 2
En caso de que el Anion Gap de AUMENTADO (+ de 14) Entonces estamos viendo una ACIDOSIS
METABOLICA POR EXCESO DE ACIDO...
Si el AG DA NORMAL, entonces, el paciente tiene un AC METAB. POR FALTA DE BASE...
Este tipo de ac metab, también es conocida por ACIDOSIS HIPERCLOREMICA
CASO CLINICO
A 25-year-old woman presents with nausea
and vomiting of 2 days duration. She is not
on any medications and was previously well
until now. Her physical examination is normal
except for a postural drop in her blood pressure
from 110/80 mm Hg supine to 90/80 mm Hg
standing. Her serum electrolytes are sodium
130 mEq/L, potassium 3 mEq/L, chloride 90
mEq/L, bicarbonate 30 mEq/L, urea 50 mg/dL,
and creatinine 0.8 mg/dL. Which of the following
electrolytes is most likely to be filtered
through the glomerulus but unaffected by
tubular secretion?
(A) potassium
(B) sodium
(C) bicarbonate
(D) urea
(E) creatinine
(D) Urea is filtered at the glomerulus, and
thereafter, any movement in or out of tubules
is a passive process depending on gradients,
not secretion. Reabsorption of urea in the distal
tubule and collecting duct, when urine flow is
reduced, results in the disproportionate elevation
of urea nitrogen over creatinine in prerenal
azotemia.
SODIO
• 1. Cuales son las causas de Hiponatremia?
• R = Es debida a múltiples patologías como SÍNDROME DE SECRECIÓN INAPROPIADA DE HORMONA ANTIDIURETICA o cirrosis hepática, IC.
• 2. Cual es el cuadro clínico de la hiponatremia?
• R = Cefalea, calambres, fatiga, letargia, APATIA, ANOREXIA, DESORIENTACION.
• 3. Como realizas el diagnostico de origen de hiponatremia?
• R = ES en sangre, NA URINARIO < 20 MEQ es debido a perdidas EXTRARENALES y NA URINARIO > 20 MEQ son de causa RENAL
• 4. Cual es la etiología de la hipernatremia?
• R = Es debida a PERDIDA DE AGUA o retención de sodio.
• 5. Cual es el cuadro clínico de la hipernatremia?
• R = 1) DEPLECION DE VOLUMEN: Como hipotensión ortostatica, taquicardia, disminución de la turgencia de la piel. 2) VOLUMEN NORMAL:
Datos neurológicos como IRRITABILIDAD, CONFUSION , letárgia, HIPERREFLEXIA.
• 6. Cual es el manejo de la hipernatremia?
• R = DIURÉTICOS y solución glucosada al 5%
• 7. Que puede causar hiponatremia isotónica?
• R = HIPERLIPIDEMIA, hiperproteinemia
• 8. Que causa hiponatremia hipertónica?
• R = HIPERGLUCEMIA, manitol, sorbitol, radiocontraste
• 9. Que fármaco se utiliza para la hiponatremia que provoca el sx cerebral con perdida de sal?
• R = FLUDOCORTISONA
SIADH
• 10. Que tumoración cancerosa provoca sx de secreción inapropiada de ADH?
• R = Carcinoma pulmonar de CÉLULAS PEQUEÑAS en pulmón
• 11. Que caracteriza al síndrome de secreción inapropiada de ADH?
• R = HIPONATREMIA, osmolaridad <280, OSMOLARIDAD URINARIA >150, función tiroidea y suprarrenal normal,
NA URINARIO > 20.
• -A 65-year-old woman is admitted to the hospital after a stroke resulting in left hemiparesis. She has no trouble
swallowing and is on a full diet. One week later, on routine biochemistry, her sodium is 128 mEq/L and
osmolality is 270 mOsm/kg. She has no symptoms and is euvolemic on clinical examination. Urine sodium is 40
mEq/L and urine osmolality is 450 mOsm/kg.
• -SIADH is associated with many CNS diseases including meningitis, encephalitis, tumors, trauma, stroke, and
acute porphyria. It is assumed that antidiuretic hormone (ADH) in these patients is secreted in response to
direct stimulation of the hypothalamic osmoreceptors.
• -A 42-year-old man is on amitriptyline for depression. Recently he has been feeling more lethargic and unwell.
His clinical examination is normal. Serum sodium is 125 mEq/L and osmolality is 260 mOsm/kg, and urine
sodium is 40 mEq/L and osmolality is 450 mOsm/kg:
• -Amitriptyline is one of the psychoactive drugs that cause SIADH. Others include phenothiazines, serotonin
reuptake inhibitors, and monoamine oxidase inhibitors (MAOIs). Antineoplastic drugs such as vincristine and
cyclophosphamide also cause SIADH, as does the hypoglycemic agent chlorpropamide
• 12. Que fármaco inhibe la acción de la ADH?
• R = DEMECLOCICLINA que actúa sobre receptor V2 de la ADH.
CASO CLINICO
A 77-year-old man with a mass in the lung
develops asymptomatic hyponatremia. His JVP
is 4 cm, heart sounds are normal, and the lungs
are clear. The urine sodium is 64 mEq/L and
osmolality 550 mOsm/kg. Which of the following
is the most likely diagnosis?
(A) nephrotic syndrome
(B) syndrome of inappropriate antidiuretic
hormone (SIADH) production
(C) renal metastases from lung cancer
(D) lung metastases from hypernephroma
(E) renal tubular acidosis (RTA)
(B) The urine osmolality in patients with
SIADH need not be hypertonic to plasma, but
only inappropriately high compared with
serum. The major characteristics of SIADH
include hyponatremia, volume expansion
without edema, natriuresis, hypouricemia, and
normal or reduced serum creatinine level, with
normal thyroid and adrenal function. (Kasper,
pp. 2102–2103)
POTASIO• 13. Cuales son las causas de la hipokalemia?
• R = Incremento en el recambio celular, disminución de la ingesta o un aumento de su excreción
• 14. Que medida de laboratorio es útil para el diagnostico de hipokalemia de origen renal o extrarenal?
• R = EL GRADIENTE TRANSTUBULAR DE POTASIO (TTKG), el cual si es < 2 TRADUCE HIPOKALEMIA NORENAL y UNO >
10 ES DEBIDO A PERDIDAS RENALES. EL K URINARIO <25 MEQ/DIA es NO RENAL y K URINARIO > 30
MEQ/DIA traduce perdida de ORIGEN RENAL.
• 15. Cuales son lo síntomas de hipokalemia?
• R = DEBILIDAD MUSCULAR, fatiga, CALAMBRES. <2.5 Meq parálisis flácida, HIPORREFLEXIA, hipercapnia y rabdomiolisis.
• 16. Que elementos participan en la captación de potasio?
• R = La insulina en presencia de glucosa, estimulación B adrenérgica
• 17. Cual es el regulador de potasio mas importante?
• R = ALDOSTERONA facilita la excreción urinaria de K en los TCD.
• 18. Cuales son las causas de la hiperkalemia?
• R = > 5.5 mEq. DISMINUCION DE LA EXCRESION: HIPOALDOSTERONISMO, IRA o IRC, ESPIRONOLACTONA,
IECA, B bloqueador. Desplazamiento de K al interior: Rabdomiolisis, hemolisis y ACIDOSIS METABOLICA
• 19. Cual es el cuadro clínico de hiperkalemia y EKG?
• R = DEBILIDAD MUSCULAR, distención abdominal, DIARREA. EKG con onda T picuda, QT corto y depresión del
segmentoT.
• 20. Cual es el manejo de la hiperkalemia?
1) INSULINA + GLUCOSA AL 10%, SALBUTAMOL inhalado, GLUCONATO DE CALCIO IV, HCO3 que produce ingreso a la
celula de K.
2) EL TRATAMIENTO AGUDO CON GLUCONATO DE CALCIO DEBE RESERVARSEcuando se presentan manifestaciones
cardiacas de toxicidad celular por exceso de potasio (calcitosis).
CASOS CLINICOS
CASO CLINICO
A 60-year-old woman with heart failure and
normal renal function is started on furosemide
(Lasix) 80 mg/day. She notices a good diuretic
response every time she takes the medication.
Afew weeks later, she is feeling unwell because
of fatigue and muscle weakness, but her heart
failure symptoms are better. Which of the following
is the most likely explanation for her
muscle weakness?
(A) hyponatremia
(B) hypernatremia
(C) hypokalemia
(D) hyperkalemia
(E) anemia
(C) Hypokalemia can result in paralytic ileus,
rhabdomyolysis, weakness, and cardiac repolarization
abnormalities. It is a common complication
along with hyponatremia of starting
patients on diuretics. (Kasper, p. 260)
CALCIO
• 21. Cuales son las 2 principales causas de hipocalcemia?
• R = ACCIÓN INSUFICIENTE DE PTH O VIT D siendo la mas frecuente IRA o IRC
• 22. Cuales son los signos y síntomas de HIPOCALCEMIA?
• R = Calambres, TETANIA, laringoespasmo, convulsiones, SIGNO DE CHEVOSTEK Y TROUSSEAU POSITIVO.
EKG con PROLONGACIÓN DEL ST
• 23. Cual es el manejo de la hipocalcemia?
• R = En presencia de TETANIA, ARRITMIAS o CONVULCIONES se indica GLUCONATO DE CALCIO en
infusión. ASINTOMÁTICOS se da CALCIO Y VIT D
• 24. Cuales son las causas mas comunes de HIPERCALCEMIA?
• R = HIPERPARATIROIDISMO primario y provenientes de malignidad en 90%. Fármacos, IR, trasplante
renal, SINDROME DE LECHE-ALCALI (caracterizado por hipercalcemia, alcalosis, nefrocalcinosis e IR
CAUSADO POR CONSUMO EXCESIVO DE CA Y ANTIÁCIDOS).
• 25. Cuales son los síntomas de hipercalciemia?
• R = Comienza >10.4 mg/dl, ESTREÑIMIENTO, poliuria, nausea, vomito, ULCERA PÉPTICA,
debilidad, depresión, LETARGO. EXTRASÍSTOLES VENTRICULARES. EKG con
intervalo QT ACORTADO
• 26. Cual es el manejo de hipercalcemia?
• R = La hipercalcemia tiende a depletar el volumen por DM insípida nefrogenica, por lo tanto se debe
restablecer EUVOLEMIA + FUROSEMIDE. Hipercalcemia por MALIGNIDAD SE USA BIFOSFONATOS,
DIÁLISIS. Clorhidrato de Cinacalcet que suprime la secreción de PTH y disminuye calcio.
FOSFATO
• 27. Que situaciones disminuyen la absorción del TCP de fosfato?
• R = Expansión de volumen, glucocorticoides, DISFUNCIÓN TUBULAR PROXIMAL COMO EN EL SX DE
FANCONI. La hormona del crecimiento aumenta su reabsorción.
• 28. Cuales son las causas de hipofosfatemia?
• R = Fármacos fosfaturicos: XANTINAS, TEOFILINA, esteroides. HIPERPARATIROIDISMO E HIPERGLUCEMIA.
Alcalosis respiratoria, ADMINISTRACIÓN DE INSULINA Y GLUCOSA SIMULTÁNEOS y
SÍNDROME DE HUESO HAMBRIENTO (posterior a para tiroidectomía para corrección de hiperparatiroidismo, el
deposito masivo de fosforo y calcio dan como resultado hipocalcemia y fosfatemia)
• 29. A que se debe la debilidad muscular o rabdomiolisis en hipofosfatemia?
• R = Aumento de la afinidad de la Hb al O2 por difosfoglicerato en el eritrocito, por lo tanto, disminuye la
oxigenación tisular y metabolismo celular.
• 30. Cual es el manejo de la hipofosfatemia?
• R = Si se administra FOSFATO RÁPIDAMENTE puede OCASIONAR HIPERCALCEMIA y SE PREFIERE VO.
• 31. Cuales son las causas más comunes de hiperfosfatemia?
• R = HIPOPARATIROIDISMO, IR, excreción renal de fosforo disminuida, aumento de fosforo al espacio EC y
administración de sales de fosforo o Vitamina D.
• 32. Cual es el cuadro clínico de la hiperfosfatemia?
• R = Esta RELACIONADO A LA HIPERCALCEMIA
• 33. Cual es el manejo de la hiperfosfatemia?
• R = RESTRICCIÓN de fosforo en dieta, QUELANTES de fosforo en IR como SEVELAMER O ALOGLUTAMOL.
MAGNESIO
• 34. Cuales son las causas de hipomagnesemia?
• R = Aumento del volumen intravascular, DIURÉTICO, cisplatino, aminoglucósido y anfotericina B.
• 35. Cuales son los signos y síntomas de hipomagnesemia?
• R = DEBILIDAD, CALAMBRES, temblor, movimiento atetoide, arritmias ventriculares, confusión.
• 36. Cuales son los datos de lab que te indican hipomagnesemia?
• R = Excreción urinaria es mas confiable, >10-30 mg/dl. La HIPOMAGNESEMIA SIEMPRE SE ASOCIA A
HIPOCALCEMIA E HIPOKALEMIA. EKG intervalo QT PROLONGADO. Supresión de la PTH.
• 37. Cual es el manejo de la hipomagnesemia?
• R = Mg IV o IM. OXIDO DE Mg VO.
• 38. Cuales son las causas de hipermagnesemia?
• R = IR o ingesta de Mg
• 39. Cuales son los síntomas de hipermagnesemia?
• R = Debilidad muscular, DISMINUCIÓN DE ROTS y confusión. RUBOR FACIAL, bradiarritmias.
• 40. Cuales son las manifestaciones EKG de hipermagnesemia?
• R = Aumento del INTERVALO PR, ENSANCHAMIENTO DEL QRS y ondas T ACUMINADAS.
• 41. Cual es el manejo de la hipermagnesemia?
• R = Ca en IRC, el cual actúa como antagonista
BENCE JONES
• 42. Que cifras de urianalisis en 24 hrs revela proteinuria?
• R = >150 mg/24 hrs
• 43. Que enfermedad se relaciona con la proteinuria de Bence Jones?
• R = Mieloma múltiple
• -A 67-year-old man presents with symptoms of renal colic. Plain x-rays of the abdomen reveal no obvious stone. An intravenous pyelogram
(IVP) is ordered to confirm the clinical diagnosis. Which of the following coexisting medical conditions increases the risk of contrast-induced
nephropathy?
• (A) hyperparathyroidism
• (B) pyelonephritis
• (C) nephrolithiasis
• (D) hypernephroma
• (E) multiple mieloma
• -The danger of ARF after IVP has led to caution, especially in patients with multiple myeloma. The patient should not be dehydrated if IVP is
necessary. The risk is also increased in patients with diabetes mellitus or chronic renal failure.
• -A74-year-old man presents with fatigue, shortness of breath on exertion, and back and rib pain, which is made worse with movement.
Investigations reveal he is anemic, calcium, urea, and creatinine are elevated. X-rays reveal multiple lytic lesions in the long bones and ribs,
and protein electrophoresis is positive for an immunoglobulin G (IgG) paraprotein. Which of the following is the most likely mechanism for
the renal injury?
• (A) plasma cell infiltrates
• (B) tubular damage by light chains
• (C) glomerular injury
• (D) vascular injury by light chains
• (E) uric acid crystals
• -In multiple myeloma, tubular damage by light chains is almost always present. The injury is a direct toxic effect of the light chains or
indirectly from the inflammatory response. Infiltration by plasma cells and glomerular injury is rare. Hypercalcemia may produce transient
or irreversible renal damage as do amyloid and myeloma cell infiltrates.
IRA• 44. Cual es la causa mas común de IRA?
• R = Azoemia prerrenal por HIPO PERFUSIÓN
• 45. A que se debe el daño renal ocasionado por la toma persistente de AINES?
• R = Hay aumento en la proporción de oxido nítrico, BLOQUEO DE LA PRODUCCIÓN DE PROSTAGLANDINAS, inhibición de los leucotrienos,
producción elevada de glucoproteina IIb/IIIa y disminución en la producción de renina.
• 46. Como se encuentra en la AZOEMIA PRERRENAL la relación BUN/ Creatinina?
• R = > 20
• 47. Como se encuentra el FENA en la IRA?
• R = Prerenal <1% y postrenal > 2%.
• Cual de las siguientes patologías constituyen indicacion de dialisis precoz en IRA?
• R = Hiperkalemia refractaria al tratamiento, sobrecarga de volumen no manejable con tratamiento medico, ACIDOSIS METABOLICA
INCONTROLABLE y pericarditis urémica. La IRA por contraste NO es una indicación.
• Cuales hallazgoz analiticos son comunes en IRA?
• Hiperkalemia, acidosis metabólica, hiperfosfatemia e hipermagnesemia
• -A 46-year-old woman with nausea and vomiting presents to hospital because of lightheadedness when standing and decreased urine
output. She looks unwell; the blood pressure supine is 90/60 mm Hg and 80/60 mm Hg when standing. Her abdominal, heart, and lung
examinations are normal. Which of the following laboratory values suggests prerenal azotemia in this patient?
• (A) markedly elevated urea, unchangedvcreatinine
• (B) unchanged urea, elevated creatinine
• (C) little change in either creatinine or ureavfor several days after oliguria develops
• (D) urea/creatinine ratio of 10
• (E) urea/creatinine ratio of 25
• -The ratio of BUN/creatinine is usually <10–15 in intrinsic renal disease and >20 in prerenal azotemia.
• -A74-year-old man from a nursing home is not feeling well and is confused. He is not able to give any reliable history. His serum sodium is
120 mEq/L and osmolality is 265 mOsm/kg.
• -The combination of ECF volume contraction with high urinary sodium (20 mmol/L) suggests renal fluid loss. This is commonly caused by
diuretics or glucosuria.
CASO CLINICO
A 67-year-old man presents with symptoms of
renal colic. Plain x-rays of the abdomen reveal no
obvious stone. An intravenous pyelogram (IVP)
is ordered to confirm the clinical diagnosis.
Which of the following coexisting medical conditions
increases the risk of contrast-induced
nephropathy?
(A) hyperparathyroidism
(B) pyelonephritis
(C) nephrolithiasis
(D) hypernephroma
(E) multiple myeloma
(E) The danger of ARF after IVP has led to caution,
especially in patients with multiple
myeloma. The patient should not be dehydrated
if IVP is necessary. The risk is also
increased in patients with diabetes mellitus or
chronic renal failure. (Kasper, pp. 1646–1647)
OSTEODISTROFIA RENAL
NTA
• 48. A que padecimiento se le conoce como NTA?
• R = IRA por lesiones tubulares siendo las principales causas la isquémica y por
toxinas
• 49. Que nefrotoxinas endógenas ocasionan NTA?
• R = Productos contenedores de HEM, ACIDO ÚRICO, MIOGLOBINURIA por
RABDOMIOLISIS
• 50. Cuales son los datos de lab de NTA?
• R = ORINA COLOR CAFÉ LODOSO por Hb y cilindros epiteliales de túbulos renales
• 51. Cual es el manejo de la NTA?
• R = Se debe prevenir hipercalcemia, FUROSEMIDE, RESTRICCIÓN DE PROTEÍNAS
DIETÉTICAS Y DIÁLISIS EN PACIENTE CRITICO
• Cual es la complicación mas común y mas temida de la NTA?
• R = Falla cardiaca
CASO CLINICO
A 74-year-old woman develops acute sepsis
from pneumonia and is admitted to the intensive
care unit because of hypotension. She is
started on antibiotics, and her blood pressure is
supported with intravenous normal saline.
Despite this she remains oliguric and develops
ARF. Her urinalysis has heme-granular casts
and the urine sodium is 56 mEq/L. Which of
the following is the most likely cause of her
ARF?
(A) nephrotoxic antibiotics
(B) acute infectious GN
(C) acute tubular necrosis (ATN)
(D) contrast nephropathy
(E) cholesterol emboli
(C) ATN is a common complication of prolonged
hypotension and ischemic injury to the
renal tubules. Heme-granular “muddy brown”
casts are consistent with ATN. Peritoneal dialysis
is preferred with cerebral trauma as well as
with severe heart failure because of risk of
hemorrhage or hypotension with hemodialysis.
In particular, peritoneal dialysis does not
require any anticoagulation and is safer if head
trauma has occurred. (Kasper, pp. 1645–1646)
NEFRITIS INSTERSTICIAL AGUDA
• 52. Que es la nefritis intersticial aguda?
• R = Respuesta inflamatoria intersticial con edema y posible daño celular
• 53. Cuales son las causas de nefritis intersticial?
• R = FÁRMACOS 70%. Infecciosas e inmunitarias
• 54. Cuales son los datos clínicos de nefritis intersticial aguda?
• R = Se caracteriza por un DETERIORO ABRUPTO DE LA FUNCIÓN RENAL
acompañado de poliuria, Nicturia, ACIDOSIS METABOLICA y glucosuria. FIEBRE90%,
ARTRALGIAS80%, EXANTEMA20%. Bh: EOSINOFILIA EN FASE AGUDA. EGO con
EOSINOFILURIA, hematuria, leucocituria y cilindros leucocitarios.
• 55. Cuales son los hallazgos en la biopsia con nefritis intersticial aguda?
• R = Infiltración de células inflamatorias en intersticio renal con edema, predominan mononucleares y LT,
GRANULOMAS NO CASEIFICANTES.
• 56. Cual es el manejo de la nefritis intersticial aguda?
• R = RETIRAR EL FACTOR predisponente. Puede llegar a REQUERIR DIÁLISIS, MEDIDAS DE SOSTEN,
METILPREDNISOLONA en NI POR FÁRMACOS
• 57. Que caracteriza a la necrosis intersticial crónica?
1. Ausencia de proteinuria e hipoalbuminemia
2. Piuria estéril y leucocitosis mas que hematuria
3. Poliuria y Nicturia
4. Otros defectos tubulares como ATR y osteomalacia.
GLOMERULONEFRITIS
• 58. Cuales son las causas de glomerulonefritis?
• R = Nefropatía por IgA (enf. De BERGUER), autoinmunitaria o
infecciosa.
• 59. Cuales son los datos clínicos de glomerulonefritis?
• R = HIPERTENSIÓN, EDEMA periorbitario o escrotal y
PROTEINURIA.
• 60. Cuales son los hallazgos de lab en GN?
• R = Hematuria, PROTEINURIA, cilindros leuco-eritrocitarios.
Complemento C3,C4 CH50. Ac vs MBG, ANCA
• 61. Cual es el manejo de la GN?
• R = Prednisona
IRC
• 62. Cual es la clasificación de la IRC?
• ESTADIO FG
• I 90
• II 60 – 89
• III 30 – 59
• IV < 15 O DIALISIS
• 63. Cual es el manejo restrictivo de IRC?
• R = Restricción de proteínas, restricción de Na y H2 O, restricción de Mg
entre otros.
• Que pasa con la capacidad de dilución y de concentración en la IRC?
• La capacidad de DILUCIÓN se deteriora antes que la de CONCENTRACIÓN.
SX NEFRITICO Y NEFROTICO
• 64. Que caracteriza al síndrome nefrítico?
• R = Proteinuria < 3.5 GR/ DÍA, Daño glomerular que ocasiona
HEMATURIA, edemas peri orbitarios, escrotal e HIPERTENSION.
• 65. Cuales son los datos de lab del sx nefrítico?
• R = HEMATURIA con eritrocitos dismorficos y CILINDROS
HEMATICOS, se solicitan ANCA, Ac vs MBG Y C3.
• 66. Cual es el manejo del síndrome nefrítico?
• R = DISMINUIR LA TA, corregir causa de fondo, restricción de H20-
diuréticos- diálisis, METILPREDNISOLONA.
• 67. Que entidades patológicas producen sx nefrotico?
• R = DM, amiloidosis y LES. Presentándose como GLOMERULONEFRITIS MEMBRANOSA EN ADULTOS y
ENFERMEDAD DE CAMBIOS MÍNIMOS EN NIÑOS.
• -A 64-year-old man presents with weight gain, shortness of breath, easy bruising, and leg swelling. On examination, his blood
pressure is 140/80 mm Hg, pulse 100/min, JVP 4 cm, heart sounds normal, and lungs are clear. There is a 3+ pedal and some
periorbital edema. Investigations include a normal chest x-ray (CXR), electrocardiogram (ECG) with low voltages, anemia, high
urea and creatinine, and 4 g/day of protein in the urine. A renal biopsy, which shows nodular deposits that have an apple-green
birefringence under polarized light when stained with Congo red. Which of the following is the most likely diagnosis?
• (A) amyloidosis
• (B) multiple myeloma
• (C) diabetic nephropathy
• (D) minimal change disease
• (E) immunoglobulin A (IgA) nephropathy
• - Renal amyloidosis can be primary (AL) or secondary amyloidosis (AA). The hallmark finding, nephrotic syndrome, is present in
25% of patients at presentation and probably develops ultimately in over 50%. The apple-green birefringence deposits under
polarized light are diagnostic of amyloidosis, and not seen in any other renal disease.
• -A 42-year-old man notices leg and facial swelling but no other symptoms. His examination is pertinent for 3+ pedal edema
including periorbital edema. A 24-hour urine collection reveals 5 g of proteinuria. Which of the following is the most likely
diagnosis?
• (A) sickle cell disease
• (B) medullary sponge kidney
• (C) radiation nephritis
• (D) staphylococcal infection
• (E) amyloid disease
• -In addition to amyloid disease, other conditions associated with nephrotic syndrome are secondary syphilis, malaria, and
treatment with gold salts. Minimal change nephrotic syndrome, focal glomerular sclerosis, membranous nephropathy, and
membranoproliferative GN are the primary renal diseases that present as ephritic syndrome.
• 68. Cuales son los signos y síntomas del síndrome nefrótico?
• R = EDEMA periférico por albumina > 3 GR/DL que puede
llegar a anasarca.
• 69. Cuales son los datos de lab del síndrome nefrótico?
• R = EGO: en racimos de uvas por LIPIDURIA, PROTEINURIA
>3.5 gr/dl/ 24 hrs. QS: ALBUMINA <3 GR/ DL, proteínas
totales > 6 gr/dl , HIPERLIPIDEMIA.
• 70. Cual es el manejo del síndrome nefrótico?
• R = PREDNISONA, restricción de Na para controlar edema,
diuréticos. Hay riesgo de IRC con proteinuria > 5 g/24 hrs.
• 71. De que manera afecta la coagulación el sx nefrótico?
• R = ESTADO HIPERCOAGULABLE cuando la ALBUMINA SE
ENCUENTRA <2 GR/DL, se requiere de anti coagulación
CASO CLINICO
A 64-year-old man presents with weight gain,
shortness of breath, easy bruising, and leg
swelling. On examination, his blood pressure is
140/80 mm Hg, pulse 100/min, JVP 4 cm, heart
sounds normal, and lungs are clear. There is
a 3+ pedal and some periorbital edema.
Investigations include a normal chest x-ray
(CXR), electrocardiogram (ECG) with low voltages,
anemia, high urea and creatinine, and
4 g/day of protein in the urine. Arenal biopsy,
which shows nodular deposits that have an
apple-green birefringence under polarized light
when stained with Congo red. Which of the
following is the most likely diagnosis?
(A) amyloidosis
(B) multiple myeloma
(C) diabetic nephropathy
(D) minimal change disease
(E) immunoglobulin A (IgA) nephropathy
(A) Renal amyloidosis can be primary (AL) or
secondary amyloidosis (AA). The hallmark
finding, nephrotic syndrome, is present in 25%
of patients at presentation and probably develops
ultimately in over 50%. The apple-green
birefringence deposits under polarized light
are diagnostic of amyloidosis, and not seen in
any other renal disease. (Kasper, p. 1689)
CASO CLINICO
A 42-year-old man notices leg and facial swelling
but no other symptoms. His examination is pertinent
for 3+ pedal edema including periorbital
edema. A24-hour urine collection reveals 5 g of
proteinuria. Which of the following is the most
likely diagnosis?
(A) sickle cell disease
(B) medullary sponge kidney
(C) radiation nephritis
(D) staphylococcal infection
(E) amyloid disease
(E) In addition to amyloid disease, other conditions
associated with nephrotic syndrome are
secondary syphilis, malaria, and treatment
with gold salts. Minimal change nephrotic syndrome,
focal glomerular sclerosis, membranous
nephropathy, and membranoproliferative GN
are the primary renal diseases that present as
nephrotic syndrome. (Kasper, p. 1689)
GLOMERULONEFRITIS RP Y PI
• 72. Que distingue a la GN rápidamente progresiva?
• R = Es una situación clínica en la cual el daño glomerular es tan agudo que
la función renal se deteriora en días o semanas de manera rápida y
progresiva y también cuando >50% DE LOS GLOMÉRULOS contienen
FORMAS SEMILUNARES en la BIOPSIA.
• 73. Cual es el manejo de la GN rápidamente progresiva?
• R = METILPREDNISOLONA sola combinada con CICLOFOSFAMIDA
• 74. Que enfermedades mas comúnmente preceden a la GN infecciosa?
• R = IMPÉTIGO y FARINGITIS.
• 75. Cual es el cuadro clínico de la GNPI?
• R = Oliguria, edema e hipertensión
CASO CLINICO
A 24-year-old woman presents with nausea,
vomiting, anorexia, and gross hematuria. She
had a sore throat 2 weeks ago that resolved on
its own. On examination, her blood pressure is
160/90 mm Hg, pulse 90/min, JVP is 7 cm,
heart sounds are normal, there is 1+ pedal
edema, and the lungs are clear. She has a renal
biopsy. Which of the following electron
microscopy findings on the renal biopsy is
most likely in keeping with poststreptococcal
GN?
(A) diffuse mesangial deposits
(B) no deposits
(C) electron-dense endothelial deposits
(D) closed capillary lumen
(E) subepithelial humps
(E) These humps are discrete, electron-dense nodules
that persist for about 8 weeks and are highly
characteristic of the disease. Light microscopy
reveals diffuse proliferation, and immunofluorescence
reveals granular immunoglobulin
G (IgG) and C3. Most patients will recover
spontaneously. (Kasper, pp. 1680–1681
CASO CLINICO
A 19-year-old man presents with malaise,
nausea, and decreased urine output. He was
previously well, and his physical examination
is normal except for an elevated jugular venous
pressure (JVP) and a pericardial rub. His electrolytes
reveal acute renal failure (ARF). Which
of the following findings on the urinalysis is
most likely in keeping with acute glomerulonephritis
(GN)?
(A) proteinuria
(B) white blood cell casts
(C) granular casts
(D) erythrocyte casts
(E) hyaline casts
(D) Both granular and erythrocyte casts are present,
but the latter indicate bleeding from the
glomerulus and are most characteristically seen.
Red cells reach the urine probably via capillary
wall “gaps” and form casts as they become
embedded in concentrated tubular fluid with
high protein content. Proteinuria is invariably
present but is not as specific. (Kasper, pp. 250–251)
CRIOGLOBULINEMIA
• 76. Que caracteriza a la crioglobulinemia y con que enfermedades se relaciona?
• R = Es una vasculitis de pequeños vasos DEBIDO A INMUNOGLOBULINAS que se
PRECIPITAN CON EL FRIO relacionándose mucho con el VHC +++ mas
que con el VHB, endocarditis o LES.
• 77. Cual es el cuadro clínico de la crioglobulinemia?
• R = PURPURA 90 %, ARTRITIS 80% , Neutropenia 70 %, DANO RENAL 50% (de este
el 80% se manifiesta como GLOMERULONEFRITIS
MEMBRANOPROLIFERATIVA Y TROMBOS EN EL INTERIOR DE LOS
CAPILARES GLOMERULARES)
• 78. Cual es el manejo de la crioglobulinemia?
1) Proteinuria y daño renal leve dándose TRATAMIENTO SINTOMÁTICO Y ESPECIFICO
2) PARA VHC SI ES QUE ESTA ASOCIADO con origen se da INTERFERON Y RIVABIRINA,
3) Si la PROTEINURIA se encuentra en RANGO NEFRÓTICO se da
METILPREDNISOLONA, PLASMAFERESIS O RITUXIMAB.
NEFROPATÍA POR IgA O SX DE
BERGUER
• 79. Que caracteriza a la nefropatía por IgA o sx de Berguer?
• R = Se deposita IgA en el mesangio glomerular, observándose la MISMA LESIÓN
EN PURPURA DE HENOCH- SCHONLEIN
• 80. Cuales son los signos de nefropatía por IgA?
• R = HEMATURIA “SINFARINGITICA” con orina color COCA-COLA 100%. IVRS50%, GI 10%
• 81. Que datos se encuentran en la biopsia en la nefropatía por IgA ?
• R = DEPOSITOS DE IgA acompañados de C3 E IgG.
• 82. Cual es el manejo de la nefropatía por IgA?
1) IECAS, ESTEROIDES.
2) En caso de proteinuria se combina CICLOFOSFAMIDA CON AZATRIOPINA.
• 83. Cual es el pronóstico de nefropatía por IgA?
• R = BUENO, pero es MALO para los que desarrollan SEMILUNAS EN LA BIOPSIA
GN PAUCINMUNITARIA
• 84. En que enfermedad se presenta GN paucinmunitaria?
1) GRANULOMATOSIS DE WEGENER,
2) ENFERMEDAD DE CHURG- STRAUSS,
3) POLIANGEITIS MICROCITICA.
• 85. Que patogénesis se encuentra hubicada en 80% de las
glomerulonefritis paucinmunitarias?
• R = Asociada a ANCA
• 86. Cuales son los datos de laboratorio de la GN paucinmunitaria?
• R = Patron citoplasmático c-ANCA, patrón perinuclear p-ANCA.
• 87. Cual es el manejo de la GN paucinmunitaria?
• R = ESTEROIDES, CICLOFOSFAMIDA.
AFECCION RENAL X ENF SISTEMICA
NEFROPATIA LUPICA
NEFROPATIA DIABETICA
ENFERMEDAD DE CAMBIOS MÍNIMOS
• 88. Cual es la etiología de la enfermedad de cambios mínimos?
• R = IDIOPÁTICO. MAS FRECUENTE EN NIÑOS. Hay una teoría que implica desorden de los linfocitos T. Se ha
encontrado asociación con procesos virales/ parasitarios y uso de AINES, amitriptilina y RIFAMPICINA.
• -A 33-year-old man from Southeast Asia, without HIV infection, is diagnosed as having pulmonary tuberculosis.
He is started on multiple medications, including INH and rifampin. Three months later, he has developed
edema. Liver tests are normal, and serum creatinine is increased by 30% over baseline. Urinalysis reveals 4+
proteinuria. A renal biopsy is performed:
• -Rifampin can cause minimal change disease as well as more severe renal damage. The described case is typical
for minimal change GN with nephrotic syndrome. Drug-induced minimal change GN frequently has an
associated interstitial nephritis. INH is not usually associated with renal disease.
• 89. Cuales son los signos y síntomas de la enfermedad de cambios mínimos?
• R = Manifestaciones del SÍNDROME NEFRÓTICO
• 90. Que datos histológicos presenta la enfermedad de cambios mínimos?
• R = A la microscopia NO REVELA LESIONES GLOMERULARES o solo una mínima prominencia mesangial
segmentaria y focal.
• 91. Cual es el tratamiento de elección en la enfermedad de cambios mínimos?
• R = ESTEROIDES, CICLOFOSFAMIDA en caso de responder a esteroides hasta completar 12 SEMANAS
NEFROPATÍA MEMBRANOSA
• 92. Que es la nefropatía membranosa?
• R = Enfermedad glomerular por DEPOSITO DE IgG y complemento constituyendo la forma mas COMÚN DE
SÍNDROME NEFRÓTICO EN ADULTOS, es idiopático 70% y el resto se debe a virus, neoplasias, drogas o
enfermedades autoinmunes
• 93. Cual es el cuadro clínico de la nefropatía membranosa?
• R = SÍNDROME NEFRÓTICO.
• 94. Que etiología tiene la NM secundaria?
• R = LES, VHC Y ENDOCARDITIS son las mas comunes. Hepatitis B, sífilis, cáncer, penicilamina y captopril.
• 95. Cuales son los signos y síntomas de NM secundaria?
• R = Relacionados a SX NEFRÓTICO
• 96. Que revela la biopsia e inmunohistoquimica en la GMN?
• R = HISTOLOGIA: PROLIFERACIÓN EN FORMA DE PARCHES, infiltrado leucocitario intraglomerular y necrosis
intracapilar. INMUNOHISTOQUIMICA: IgG siempre presente, IgM e IgA.
• 97. Cual es el manejo de la nefropatía membranosa?
• R = PROTEINURIA < 4 GR DAS IECAS CON META DE TA 125/75. PROTEINURIA > 4 GR y < 8 GR SIN
DETERIORO DE LA FUNCIÓN RENAL EL TRATAMIENTO SE ENCAMINA A DISMINUIR LA PROTEINURIA CON IECAS Y
OBSERVAR POR 6M.
GLOMERULONEFRITIS FOCAL Y
SEGMENTARIA
• 98. Cual es la etiología de la Glomerulonefritis focal y segmentaria?
• R = Primaria (idiopática) o secundaria debido a otras patologías como agenesia renal, ENFERMEDAD DE
CÉLULAS FALCIFORMES o por VIH.
• 99. Dentro de las diferentes variedades de Glomerulonefritis focal y segmentaria cuales producen mas
proteinuria?
• R = LA VARIEDAD CELULAR Y COLAPSANTE en el 80% de los casos de síndrome nefrótico
• 100. Cual es el cuadro clínico de la Glomerulonefritis focal y segmentaria?
• R = PROTEINURIA asintomática o síndrome nefrótico completo
• 101. Cuales son los datos de laboratorio y biopsia de la Glomerulonefritis focal y segmentaria?
• R = La PROTEINURIA puede oscilar DESDE <1 GR HASTA 20-30 GR y en la biopsia los hallazgos de
HISTOLOGÍA COMO SON LA VARIEDAD CELULAR Y COLAPSANTE.
• 102. Cual es el manejo de la Glomerulonefritis focal y segmentaria?
1) LOS QUE TENGAN PROTEINURIA SUBNEFROTICA 2-3 GR/ no se les dará tratamiento inmunosupresor, siendo
el principal tratamiento el control de la TA CON META DE 130/80, UTILIZANDO IECAS, hipolipemiantes,
DIURÉTICOS EN CASO DE EDEMA.
2) En los pacientes que se presentan con SÍNDROME NEFRÓTICO SE USA PREDNISONA, CONTROL DE LA TA
CON META DE 125/75, ANTICOAGULANTES (>10 GR/DIA) y en caso de RESISTENCIA SE USARA
CICLOSPORINA.
• 103. Cual es el pronostico de la Glomerulonefritis focal y segmentaria?
• R = MALO SI LA PROTEINURIA ES > 10 GR/DIA
ENFERMEDAD DE LIDDLE
• 104. Que es la enfermedad de Liddle?
• R = AD. El síndrome de Liddle (o SEUDOHIPERALDOSTERONISMO)
es un trastorno AD caracterizado por HIPERTENSIÓN SENSIBLE A
SAL con expansión de volumen, HIPOKALEMIA y ALCALOSIS
METABOLICA, con ACTIVIDAD DE RENINA Y ALDOSTERONA
EXTREMADAMENTE BAJAS y FG NORMAL.
• 105. Cual es el cuadro clínico del síndrome de Liddle?
• R = Cansancio, poliuria, polidipsia y cefalea.
• 106. Cual es el manejo del síndrome de Liddle?
• R = TRIAMTERENO y RESTRICCIÓN NA CON SUPLEMENTOS DE K
SÍNDROME DE GITELMAN
• 107. Que es el síndrome de Gitelman?
• R = Se caracteriza por HIPOKALEMIA e HIPOMAGNESEMIA, además de contar con
HIPOCALCIURIA, perdida de sal y ALCALOSIS METABÓLICA con
HIPERALDOSTERONISMO, HIPERRENINEMIA, asi como HIPERTROFIA E
HIPERPLASIA DEL APARATO YUXTAGLOMERULAR.
• 108. Cual es el cuadro clínico del síndrome de Gitelman?
• R = Debilidad muscular y DERMATITISinespecífica. ESPASMOS DISTALES
ocurren EN PERIODOS DE FIEBRE, vomito o diarrea. Algunos sufren
condrocalcinosis.
• 109. Cual es el principal diagnostico diferencial del síndrome de Gitelman?
• R = SINDROME DE BARTTER, donde la RESPUESTA A DIURÉTICO DE ASA es
obviamente preservada en el SÍNDROME DE GITELMAN.
• 110. Cual es el manejo del síndrome de Gitelman?
• R = SUPLEMENTO DE POTASIO Y MAGNESIO, además de TRIAMTERENO, amilorida
o espironolactona.
SÍNDROME DE BARTTER
• 111. Que es el síndrome de Bartter?
• R = AR. Se caracteriza por HIPOKALEMIA GRAVE, perdida de saly ALCALOSIS
METABÓLICAcon HIPERALDOSTERONISMO, HIPERRENINEMIA, así como
HIPERTROFIA E HIPERPLASIA DEL APARATO YUXTAGLOMERULAR. Casi TODOS
TIENEN NEFROCALCINOSIS por la hipercalciuria.
• 112. Cual es el cuadro clínico del síndrome de Bartter?
• R = Tipo I y II: Las manifestaciones tempranas incluyen POLIHIDRAMNIOS, retardo
en el crecimiento, polidipsia, deshidratación, avidez por sal y debilidad muscular. En
el tipo III: Se manifiesta en la INFANCIA similar al Sx de Gitelman. El tipo IV: TODO
LO ANTERIOR MAS SORDERA SENSORINEURAL E IRC TEMPRANA.
• 113. Cual es el manejo del síndrome de Bartter?
• R = Grandes cantidades de POTASIO CON ESPIRONOLACTONA, triamtereno o
amiloride.
LIDDLE, GITELMAN Y BARTTER
• Que es la enfermedad de Liddle?
• R = AD. El síndrome de Liddle (o SEUDOHIPERALDOSTERONISMO) es un trastorno AD caracterizado por
HIPERTENSIÓN SENSIBLE A SAL con expansión de volumen, HIPOKALEMIA y ALCALOSIS
METABOLICA, con ACTIVIDAD DE RENINA Y ALDOSTERONA EXTREMADAMENTE BAJAS y FG
NORMAL.
• Que es el síndrome de Gitelman?
• R = Se caracteriza por HIPOKALEMIA e HIPOMAGNESEMIA, además de contar con HIPOCALCIURIA,
perdida de sal y alcalosis metabólica con HIPERALDOSTERONISMO, HIPERRENINEMIA, asi como
HIPERTROFIA E HIPERPLASIA DEL APARATO YUXTAGLOMERULAR.
• Cual es el cuadro clínico del síndrome de Gitelman?
• R = Debilidad muscular y DERMATITIS inespecífica. ESPASMOS DISTALES ocurren EN PERIODOS DE
FIEBRE, vomito o diarrea. Algunos sufren condrocalcinosis.
• Que es el síndrome de Bartter?
• R = AR. Se caracteriza por HIPOKALEMIA GRAVE, perdida de sal y alcalosis metabólica con
HIPERALDOSTERONISMO, HIPERRENINEMIA, así como HIPERTROFIA E HIPERPLASIA DEL APARATO
YUXTAGLOMERULAR. Casi TODOS TIENEN NEFROCALCINOSIS por la
hipercalciuria.
• Cual es el cuadro clínico del síndrome de Bartter?
• R = Tipo I y II: Las manifestaciones tempranas incluyen POLIHIDRAMNIOS, retardo en el crecimiento,
polidipsia, deshidratación, avidez por sal y debilidad muscular. En el tipo III: Se manifiesta en la
INFANCIA similar al Sx de Gitelman. El tipo IV: TODO LO ANTERIOR MAS SORDERA SENSORINEURAL E
IRC TEMPRANA.
SÍNDROME DE FANCONI
• 114. Que es el síndrome de Fanconi?
• R = Se debe a la DISFUNCIÓN GLOBAL DEL TÚBULO PROXIMAL.
• 115. Cuantos tipos de Sx de Fanconi hay y que los caracteriza?
1) HEREDITARIO: Cistinosis, galactosemia, tirosinemia.
2) ADQUIRIDO: INTOXICACIÓN POR PLOMO O CADMIO, TETRACICLINAS
CADUCADAS, tolueno (inhalar pegamento), agentes quimioterapéuticos
como el cisplatino.
3) FANCONI LIKE: Pacientes adultos con disproteinemias como amiloidosis,
en enfermedad de cadenas ligeras y mieloma múltiple.
• 116. Cual es la anormalidad metabólica mas frecuente en el síndrome de
Fanconi?
• R = ACIDOSIS METABOLICA HIPERCLOREMICA.
SEUDOHIPERALDOSTERONISMO TIPO 1
• 117. Que es el seudohiperaldosteronismo tipo 1?
• R = AD/AR. También llamado síndrome de CHEEK PERRY
y se caracteriza POR PERDIDA DE SAL,
DESHIDRATACIÓN, presencia de hiponatremia,
HIPERKALEMIA y acidosis metabólica con
HIPERALDOSTERONISMO.
• 118. Cual es el manejo del
seudohiperparaldosteronismo tipo 1?
• R = Dieta alta en SODIO.
ATR
• 119. Cuales son las tubulopatias asociadas a alcalosis metabólica?
• R = Síndrome de CHEEK-PERRY(GITLEMAN), LIDDLE Y BARTTER
• 120. Que es la acidosis tubular renal o ATR?
• R = Grupo de entidades patológicas caracterizadas por defectos de transporte, ya sea
REABSORCIÓN DE BICARBONATO (HCO3), EXCRECIÓN DE HIDROGENIONES O AMBAS.
• 121. Donde se lleva a cabo la reabsorción de HCO3 y la excreción de H+?
• R = HCO3 EN EL TCP Y H+ EN TCD.
• 122. Que caracteriza a la ATR tipo I DISTAL O CLÁSICA?
• R = ACIDOSIS METABÓLICA CON BRECHA ANIONICA NORMAL O HIPERCLOREMICA, HIPOKALEMIA
E IMPOSIBILIDAD DE ACIDIFICAR LA ORINA A < 5.5 bajo el estimulo de acidosis metabólica
intensa.
• 123. Cuales son las bases fisiopatológicas de la ATR I?
• R = 1) Defecto secretor (DEFICIENCIA EN LA SECRECIÓN DE HIDROGENIONES), formas dominantes,
recesivas y asociadas a síndrome de Sjogren, LES, AR. 2) defecto de gradiente, en el cual hay un
reflujo de los hidrogeniones normalmente secretados a nivel distal (anfotericina B) y 3)
Incapacidad de generar o mantener una diferencia luminal negativa transepitelial distal como en la
uropatia obstructiva, enfermedad de células falciformes, hiperplasia adrenal perdedora de sal.
• 124. Como se diferencia la ATR tipo I de la tipo II?
• R = Que estos pacientes a menudo PRESENTARAN NEFROLITIASIS Y NEFROCALCINOSIS.
• 125. Como se maneja la ATR tipo I?
• R = ADMINISTRACIÓN DE ÁLCALI, SE PREFIERE CITRATO DE POTASIO
• 126. Que caracteriza a la ATR TIPO II O PROXIMAL?
• R = Puede ocurrir sola o acompañada de otros defectos tubulares como
síndrome de Fanconi y se caracteriza por una DISMINUCIÓN EN EL UMBRAL
RENAL PARA EL HCO3, el cual usualmente es de 22 mml/L en infantes y 26
mmol/L en adultos.
• 127. Que caracteriza a la ATR tipo IV hiperkalemica?
• R = Usualmente ocurre con la presencia de IR MODERADA, sin embargo la
magnitud de la HIPERKALEMIA y de la acidosis es desproporcionadamente
extensa para el nivel de IR.
• 128. Cual es la fisiopatología de la ATR tipo IV o hiperkalemica?
• R = Defecto en la AMONIOGENESIS
• 129. Cual es la principal enfermedad relacionada con la ATR tipo IV o
hiperkalemia?
• R = DM, además del uso de AINES, IECAS y ciclosporina.
INFECCION RENAL
• 132. Cual es el cuadro clínico de la pielonefritis?
• R = Aparición de FIEBRE > 38.5, escalofríos, DOLOR LUMBAR, disuria,
polaquiuria, HEMATURIA, NAUSEA, vomito, cefalea y diarrea.
• 133. Cual es el manejo de la pielonefritis aguda?
• R = AMPICILINA 1 gr + GENTAMICINA 1 gr por 21 DÍAS o TMP/SMZ 160/800
• 134. Cual es el manejo de la pielonefritis crónica?
• R = Lo mismo que para la aguda pero con duración de 3-6m
• 135. Cual es el manejo del absceso renal por S. Aureus?
• R = NAFCILINA 2 g IV c/4 hrs durante 2-6 SEMANAS o CEFAZOLINA 1 gr IV
c/6 hrs x 2-6 SEMANAS.
• 136. Cual es el manejo del absceso renal por E. Coli?
• R = AMPICILINA O CIPROFLOX CON GENTAMICINA
LESIONES VASCULARES RENALES
• 137. Cual es la causa de trombosis renal en niños y adultos?
• R = En niños puede ser aguda debido a DESHIDRATACIÓN por diarrea y/o vomito. En adultos se relaciona con
estados de HIPERCOAGUBILIDAD, sobre todo asociado a presencia de SÍNDROME NEFRÓTICO.
• 138. Cual es el cuadro clínico del tromboembolismo renal agudo?
• R = En caso de que la falla renal se acompañe de FIEBRE, dolor abdominal o en el flanco, NEFROMEGALIA,
leucocitosis, HEMATURIA y aparición o empeoramiento de una PROTEINURIA previa y la VENOGRAFIA es el
estudio de elección por que el Doppler tiene baja especificidad.
• 139. En que casos se sospecha de estenosis de la arteria renal?
• R = Se sospecha en pacientes con valvulopatias, IAM, arritmias o endocarditis
• 140. Cual es la presentación clínica de la estenosis de la arteria renal?
• R = Dolor en el flanco, FIEBRE, nausea, vomito, IRA. Se ha descrito EOSINOFILURIA en el cuadro renal agudo.
• 141. Cual es la causa mas frecuente de nefropatía isquémica?
• R = La estenosis ATEROMATOSA. Se consideran FR la edad, tabaquismo y ateroesclerosis difusa.
• 142. Cual es el cuadro clínico de una nefropatía isquémica?
• R = HIPERTENSIÓN INTENSA o de difícil control que pueden DESARROLLAR IRA AL COMIENZO CON IECAS.
• 143. Cuales son los hallazgos de laboratorio e imagen en la nefropatía isquémica?
• R = En los estudios de imagen hay ATROFIA RENAL y en laboratorio PROTEINURIA EN RANGO SUBNEFROTICO,
con actividad de la RENINA ELEVADA. Siendo el estándar de oro la ARTERIOGRAFÍA.
CASO CLINICO
A 56-year-old man is involved in a severe
motor vehicle accident. He develops ARF after
admission to hospital. One of the possibilities
for his ARF is posttraumatic renal vein thrombosis.
Which of the following findings is most
likely to suggest renal vein thrombosis?
(A) white cell casts on urinalysis
(B) heme-granular casts
(C) heavy proteinuria
(D) urine supernatant pink and tests positive
for heme
(E) specific gravity >1.020
(C) Renal vein thrombosis is associated with
heavy proteinuria and hematuria. Flank pain
and pulmonary embolism can also occur.
(Kasper, p. 1707)
SINDROME UREMICO HEMOLITICO
•En el SHE es común observar la prolongación del tiempo de?
•Sangría
NEFROANGIOESCLEROSIS
• 144. Que es la nefroangioesclerosis?
• R = Se refiere a la sustitución del parénquima renal por tejido fibrótico dependiente
de colágeno, COMÚNMENTE VISTO EN LA HTA y que afecta de manera
predominante a la vasculatura glomerular.
• 145. Cuales son los hallazgos a la biopsia renal relacionado con nefroesclerosis?
• R = Daño vascular con hipertrofia de la muscular y engrosamiento de la intima con
posterior deposito de material hialino, predominando en las arteriolas eferentes.
Glomeruloesclerosis con la consiguiente perdida de masa renal.
• 146. Como se manifiesta la nefroesclerosis maligna?
• R = TA >180/120 con daño importante a nivel de fondo de ojo y daño renal o
encefalopatía como consecuencia de la HTA.
• 147. Cual es el hallazgo en la biopsia renal relacionado a nefroesclerosis maligna?
• R = Necrosis fibrinoide de las arteriolas aferentes y endarteritis proliferativa de las
arterias interlobulillares con disminución de su luz e hipertrofia de las células
miointimales dando apariencia en CAPAS DE CEBOLLA.
HAS SECUNDARIA
• 148. Cuales son las manifestaciones clínicas del feocromocitoma?
• R = Triada de CEFALEA, RUBOR y PALPITACIONES aunada a HTA.
• 149. Cual es el cuadro clínico del hiperaldosteronismo?
• R = HTA, debilidad muscular, parestesias, cefalea, poliuria,
polidipsia y edema de miembros inferiores.
• 150. Cuales son los hallazgos de laboratorio en
hiperaldosteronismo?
• R = HIPOKALEMIA, alcalosis metabolica, HIPERNATREMIA e
hipomagnesemia.
CANCER RENAL
• 151. Cuales son los factores de riesgo para desarrollar cáncer renal?
• R = Tabaquismo, obesidad y abuso de AINES.
• 152. Cual es la estirpe etiológica mas común en el cáncer renal?
• R = CA DE CÉLULAS CLARAS asociado a deleción del brazo corto del CROMOSOMA 3.
• 153. Cual es el cuadro clínico del cáncer renal?
• R = HEMATURIA, DOLOR EN EL FLANCO Y MASA PALPABLE.
• 154. Cual es la estatificación y pronostico del cáncer renal?
• ESTADIO I: Tumoración de < 7 CM limitado a riñón con sobrevida del 95%
• ESTADIO II: Tumoración de > 7 CM limitado a riñón con sobrevida del 88 %
• ESTADIO III: Tumor DENTRO DE LA FASCIA DE GEROTA pero que involucra vasos mayores,
glándula suprarrenal o una sección linfática. Sobrevida a 5ª de 59 %
• ESTADIO IV: Tumor renal FUERA DE LA FASCIA DE GEROTA con sobrevida de 20 %
• 155. Cual es el manejo del cáncer renal?
1. NEFRECTOMÍA EN BLOQUE ES DE ELECCIÓN
2. Ablación por radiofrecuencia en tumores menores a 3 cm
3. Interferon alfa que aumenta sobrevida a 54 meses
CASO CLINICO
A 69-year-old woman presents with left flank
pain and hematuria. Physical examination suggests
a left-sided abdominal mass. Computerized
tomography (CT) scan of the abdomen reveals a
5-cm mass in the left kidney. Which of the following
laboratory abnormalities might also be
present?
(A) polycythemia
(B) thrombocytopenia
(C) hypocalcemia
(D) leukocytosis
(E) high renin hypertension
(A) This patient likely has hypernephroma
(renal cell carcinoma). Polycythemia is caused
by the production of erythropoietin-like factors.
There is no relationship to hypertension.
The tumor frequently presents as metastatic
disease. (Kasper, pp. 541–542)
CASO CLINICO
A 64-year-old man is admitted for hematuria
after slipping on an icy pavement. His physical
examination is normal. A selective angiogram
of the left kidney is shown. Which
of the following is the most likely diagnosis?
(A) renal cell carcinoma
(B) kidney contusion and laceration
(C) transitional cell carcinoma
(A) The diagnosis is renal cell carcinoma. There
is marked hypervascularity of the left kidney.
The arteries are irregular and tortuous, following
a random distribution. There are small vessels
within the renal vein that indicate the
blood supply of the neoplastic thrombosis
involving the renal vein. The kidney is enlarged
and abnormally bulbous in the lower pole. CT
scans have dramatically decreased the need for
arteriography in evaluating renal lesions.
(Kasper, pp. 541–542)
NEFROPATÍA ASOCIADA A PARASITOS
• 156. Cuales son los agentes mas implicados en la nefropatía asociada a
parásitos?
• R = PLASMODIUM Falciparum y Plasmodium Malariae
• 157. Cual es el cuadro clínico de la nefropatía asociada a parásitos?
• R = ALTERACIONES HIDROELECTROLITICAS HASTA IR (Creatinina > 2)
relacionado a formas maduras del parasito.
• 158. Cual es el tratamiento de la nefropatía asociada a parásitos?
• R = QUININA vía parenteral cuando hay resistencia a DOXICICLINA O
TETRACICLINA
• 159. Que ocasiona el P. Malariae a nivel renal?
• R = Es de curso crónico y ocasiona nefropatía por paludismo
• 160. Cual es el contenido de las células de estruvita?
• R = MG, AMONIO Y FOSFATO
ENFERMEDAD RENAL POLIQUISTICA
• 130. Cuales son los criterios diagnósticos sonográficos para la enfermedad renal poliquistica AD?
1) < 30ª con al menos 2 quistes, uni o bilaterales
2) 30 a 60ª con al menos 2 quistes en cada riñón
3) >60ª con al menos 4 quistes en cada riñón
• 131. Que cromosomas se ven implicados en la enfermedad poliquistica renal?
• R = 4en ADULTOS y 16en FETOS
• -A 30-year-old man presents with hematuria. His examination is normal except for an elevated blood
pressure of 164/94 mm Hg. An ultrasound of the kidneys reveals multiple renal cysts in both kidneys. His
father had a similar condition. Which of the following is not associated with this syndrome?
• (A) liver cysts
• (B) intracranial aneurysms
• (C) autosomal dominant inheritance
• (D) rheumatoid arthritis (RA)
• (E) progression to end-stage renal failure
• -RA is not associated with polycystic kidney disease. It has an autosomal dominant inheritance and
about 50% develop renal failure by age 60 years. Cysts are also seen in the liver (more common) and
pancreas. Also 5–10% of asymptomatic patients can have cerebral aneurysms. Renal transplantation is
utilized in end-stage renal failure (ESRF). The transplanted kidney cannot be affected by the disease.
CASO CLINICO
Ten days after a kidney transplant, a 32-yearold
man develops allograft enlargement, fever,
oliguria, and hypertension. Which of the following
is the most likely diagnosis?
(A) steroid hyperglycemia
(B) erythrocytosis
(C) hyperacute rejection
(D) acute rejection
(E) renal artery stenosis
(D) Renal scans initially show a reduction in
excretion with cortical retention. This is the most
common type of rejection. Most acute rejections
will respond to immunosuppressive agents if
diagnosed early. In contrast, immediate nonfunction
of a graft can be caused by damage to
the kidney during procurement and storage.
Such problems are becoming less frequent.
Obstruction, vascular compression, and ureteral
compression are other causes of primary nonfunction
of a renal graft. (Kasper, p. 1672)
MISCELANEAS
• El hipoaldosteronismo hiporreninemico que anomalía electrolítica da?
• R = Hiperkalemia
• El sx de Good Pasture , a que sitio se dirigen comúnmente sus autoanticuerpos?
• R = Dañan riñón o pulmón. Los anticuerpos se dirigen contra la COLAGENA IV en
membrana basal glomerular
• Fisiológicamente que sucede con la disminución de excreción de fosforo?
• R = Aumentan los niveles de Ca+ y se aumenta la PTH liberando el fosforo y por lo
tanto hay niveles aumentados de PTH con aumento en recambio de hueso y
lesiones subperiosticas.
• Cual es el FG normal?
• R = 145 ml/min
• Cuales son las manifestaciones bioquímicas que tienen en común las ATR?
• R = Acidosis metabólica hipercloremica hipopotasemica.
• Cual es la triada clásica de la glomerulonefritis?
• R = Hipertensión, edema y hematuria.
• Criterios para dialisis?
• R = ACIDOSIS SEVERA, HIPERKALEMIA, HIPERVOLEMIA, PERICARDITIS Y
ENCEFALOPATIA
• Cuando se considera oliguria?
• R = < 400 ml
• Que caracteriza a la cistitis instersticial?
• R = Dolor al llenado vesical con mejoría al vaciamiento bajo
tratamiento con Nifedipino y AINES.
• Que cantidad de espermatozoides da oligoespermia?
• R = <20,000,000
• Que enzima inhibe la finasteride?
• R = Alfa 5 reductasa
• Medicamento que utilizarías en la HPB?
• R = Alfabloqueadores (prazocin) y RTUP
• Cual es el valor normal de PSA?
• R = 0-4 ng normal >4 inflamatorio
• Cual es la estirpe etiológica mas común del cáncer vesical?
• R = De CÉLULAS TRANSICIONALES
• Como realizas el diagnostico de origen de hiponatremia?
• R = ES en sangre, NA urinario < 20 mEq es debido a perdidas EXTRARENALES y Na urinario > 20 mEq son de causa renal
• Como se encuentra el FENA en la IRA?
• R = Prerenal <1% y postrenal > 2%.
• Como son los cálculos de la relación BUN/ Creatinina con respecto a la IRA?
• R = The ratio of BUN/creatinine is usually <10–15 IN INTRINSIC RENAL disease and >20 IN PRERENAL AZOTEMIA.
• Qué medida de laboratorio es útil para el diagnostico de hipokalemia de origen renal o extrarenal?
• R = El gradiente transtubular de potasio (TTKG), el cual si es < 2 traduce hipokalemia no renal y uno > 10 es debido a perdidas
renales. EL K urinario <25 mEq/dia es no renal y K urinario > 30 mEq/dia traduce perdida de origen renal.
• Que caracteriza al síndrome de secreción inapropiada de ADH?
• R = HIPONATREMIA, osmolaridad <280, osmolaridad urinaria >150, función tiroidea y suprarrenal normal, Na urinario > 20.
• Cual es el cambio por lesión mas común de la nefropatía diabética?
• R = GLOMERULOESCLEROSIS DIFUSA, pero la glomeruloesclerosis nodular es característica
• -A 63-year-old woman presents for routine evaluation. She has had diabetes for the past 12 years with complications of
neuropathy and retinopathy. You decide to screen her for renal complications of diabetes. Which of the following findings is not
compatible with diabetic nephropathy?
• (A) nephrotic range proteinuria
• (B) microalbuminuria
• (C) hypertension
• (D) red blood cell (RBC) casts in urine
• (E) renal tubular acidosis (RTA) type IV
• -Red cell casts are not seen in diabetic nephropathy and suggest another acute GN process. Nephrotic range proteinuria, type IV
RTA (hyporeninemic hypoaldosteronism), hypertension, and microalbuminuria are all complications of diabetic kidney disease.
• La acidosis tubulorenal a que anomalías se encuentra asociada?
• R = Resistencia a la actividad de renina y aldosterona
• -A 44-year-old woman is having symptoms of muscle weakness and fatigue. On
examination, her blood pressure is 120/80 mm Hg, pulse 80/min, JVP 4 cm, heart sounds
normal, and lungs clear. Her serum potassium level is 2.5 mEq/L, bicarbonate 18 mEq/L, and
anion gap is normal. The urine potassium is 40 mEq/L:
• -RTA types I and II cause hypokalemia with high potassium excretion (>25 mmol/L) and a low
bicarbonate in the absence of hypertension. Diabetic ketoacidosis can also result in this
constellation of findings.
•
• -A 78-year-old man is brought to the hospital because of nausea and vomiting. On
examination he appears dry, his abdomen is soft, and the JVP is not visible. His laboratory
tests reveal hypernatremia and his calculated free water deficit is approximately 3 L. In what
part of the normal kidney is most of the water reabsorbed from?
• (A collecting ducts
• (B) proximal tubule
• (C) distal tubule
• (D) ascending loop of Henle
• (E) descending loop of Henle
• Cual es el manejo de elección en sx de Good Pasteure?
• R = Plasmaferesis e imunosupresores.
• Cuales son las causas mas comunes de IRC?
• DM, HAS, Glomerulonefritis
• CC sx uremico?
• PRURITO que no cede con antihistaminicos (solo con dialisis), ESCARCHA
UREMICA, color odre, ENCEFALOPATIA, asterixis, PERICARDITIS UREMICA,
anorexia, nausea y vomito. ANEMIA, TROMBOCITOPENIA.
• EN que enfermedad se presenta mas comunmente el hiperparatiroidismo
secundario?
• R = IR
• Cuales son las manifestaciones clínicas del feocromocitoma?
• R = Triada de cefalea, rubor y palpitaciones aunada a HTA.
• Cual es el cuadro clínico del hiperaldosteronismo?
• R = HTA, debilidad muscular, parestesias, cefalea, poliuria, polidipsia y edema de
miembros inferiores.
• Cuales son las 3 manifestaciones renales asociadas a endocarditis?
1) Secundaria a inmunocomplejos presentándose como
Glomerulonefritis membranoproliferativa
2) Secundaria a medicamentos que pueden ocasionar nefritis
instersticial aguda o NTA
3) Secundaria a embolos sépticos
• Clinica a nivel renal de la endocarditis?
• R = Hematuria, cilindros eritrocitarios, HTA e IR.
• Que medicamento se utiliza en hipercalciuria por aumento de la
absorción del calcio intestinal (> 200 mg/24 hrs)?
• R = Fosfato de celulosa por que se fija al calcio e impide su absorción
• Que tamaño deben tener los cálculos urinarios para poder pasar sin
causas obstruccion?
• R = < 6 mm
• Cual es el tratamiento de primera línea de cistitis intersticial?
• R = No hay cura. Amitriptilina y nifedipino
• El 90-95% de los tumores malignos testiculares primarios son de células germinales
“seminoma o no seminoma” , a que estirpe pertenece el resto?
• R = No germinales, leydig, sertoly y gonadoblastoma.
• -An 18-year-old man is found to have metabolic alkalosis and hypokalemia. This could be
secondary to:
• -Bartter syndrome and Liddle syndrome can be inherited in an autosomal dominant fashion.
Patients with Liddle syndrome have hypertension, whereas those with Bartter syndrome do
not. In both syndromes, hypokalemia is prominent
• -A 28-year-old man presents with a kidney stone. He is married to his first cousin, and 6
months earlier, his 8-year-old son had a kidney stone as well. The most likely diagnosis is:
• -Many of the listed disorders can cause nephrolithiasis, but cystinuria is the most common
cause of stones in childhood. It is a common inborn error of amino acid transport and is
inherited as an autosomal recessive trait. The disorder affects transport of all dibasic amino
acids (lysine, arginine, ornithine, and cystine) in the kidney and the gut, but symptoms arise
from the overexcretion of cystine because it is the least soluble.
CASO CLINICO
A 15-year-old boy develops renal colic. The
stone is not recovered, but urinalysis reveals
hexagonal crystals, and a cyanide-nitroprusside
test on the urine is positive. Which of the following
is the most likely diagnosis?
(A) cystinuria
(B) thalassemia
(C) hereditary glycinuria
(D) primary hyperoxaluria
(E) sarcoidosis
(A) Cystinuria is a congenital disorder associated
with decreased tubular resorption of cystine,
arginine, ornithine, and lysine. Only cystine
is insoluble and is the cause of renal calculi. The
typical hexagonal crystals are most likely to be
seen on an acidic early-morning urine specimen.
Apositive cyanide-nitroprusside screening
test should be confirmed by chromatography.
(Kasper, p. 1714)
GLOMERULOESCLEROSIS FOCAL
• En que lesiones renales se observa
glomeruloesclerosis focal?
1) Adicción a la heroína
2) Rechazo de injerto renal
3) Reflujo vesicoureteral
4) Agenesia renal unilateral
SONDA VESICAL PERMANENTE
• Cuales son las medidas a seguir en un paciente que
presenta sonda vesical permanente?
1) Administrar ATB cuando manifiestan clínicamente una
infección
2) La bacteriuria suele desaparecer al retirar la sonda
3) Los ATB con fines profilácticos no evitan la bacteriuria
4) Los antibióticos están indicados cuando se procede a
cambiar de sonda
5) Si el paciente orina espontáneamente es preferible
usar un colector
AMINOGLUCOSIDOS
• Que factores agravan la nefrotoxicidad por
aminoglucodidos?
1) Edad avanzada
2) Tratamiento prolongado
3) Diuréticos
4) Insuficiencia renal
YODOHIPURATO 131
• La radiografia radioisotopica marcada con
yodohipurato 131 de que patología tiene
interés diagnostico?
• Hipertensión vasculorenal
Consejos
1. Leer diariamente y en bloques
2. Adiós Partys un tiempo
3. Has ejercicio y come bien durante el estudio
4. Toma algún curso bueno si tienes la posibilidad
5. Ten Fe.
BIBLIOGRAFIA
• EXARMED
• PAPADAKIS
• CTO
• HARRISON
• AMIR
• USMLE STEPS

Más contenido relacionado

La actualidad más candente

Síndrome de Gitelman (Tubulopatias)
Síndrome de Gitelman (Tubulopatias)Síndrome de Gitelman (Tubulopatias)
Síndrome de Gitelman (Tubulopatias)Daniela Grijalva
 
Nefropatía Diabética
Nefropatía DiabéticaNefropatía Diabética
Nefropatía DiabéticaJaime Cruz
 
Hemorragia digestiva baja
Hemorragia digestiva bajaHemorragia digestiva baja
Hemorragia digestiva bajaTedson Murillo
 
Urgencias: Sangrado Del Tubo Digestivo Bajo
Urgencias: Sangrado Del Tubo Digestivo BajoUrgencias: Sangrado Del Tubo Digestivo Bajo
Urgencias: Sangrado Del Tubo Digestivo BajoGabriela Bonilla
 
Absceso hepático amebiano
Absceso hepático amebianoAbsceso hepático amebiano
Absceso hepático amebianoFrancisco Vargas
 
Diarrea crónica. Diagnóstico y tratamiento. 2018
Diarrea crónica. Diagnóstico y tratamiento. 2018Diarrea crónica. Diagnóstico y tratamiento. 2018
Diarrea crónica. Diagnóstico y tratamiento. 2018Pediatriadeponent
 
Colecistitis crónica
Colecistitis crónicaColecistitis crónica
Colecistitis crónicaamo_cf
 

La actualidad más candente (20)

Apendicitis
ApendicitisApendicitis
Apendicitis
 
Síndrome de Gitelman (Tubulopatias)
Síndrome de Gitelman (Tubulopatias)Síndrome de Gitelman (Tubulopatias)
Síndrome de Gitelman (Tubulopatias)
 
Ascitis - Medicina Interna II
Ascitis - Medicina Interna II Ascitis - Medicina Interna II
Ascitis - Medicina Interna II
 
Hiperparatiroidismo
HiperparatiroidismoHiperparatiroidismo
Hiperparatiroidismo
 
Nefropatía Diabética
Nefropatía DiabéticaNefropatía Diabética
Nefropatía Diabética
 
Cirrosis hepatica
Cirrosis hepaticaCirrosis hepatica
Cirrosis hepatica
 
Comunicación ia
Comunicación iaComunicación ia
Comunicación ia
 
Patologia de la vesícula biliar y las guías de tokyio slideshare
Patologia de la vesícula biliar y las guías de tokyio slidesharePatologia de la vesícula biliar y las guías de tokyio slideshare
Patologia de la vesícula biliar y las guías de tokyio slideshare
 
Hemorragia digestiva baja
Hemorragia digestiva bajaHemorragia digestiva baja
Hemorragia digestiva baja
 
DRENAJE VENOSO ANOMALO PRESENTACION
DRENAJE VENOSO ANOMALO PRESENTACIONDRENAJE VENOSO ANOMALO PRESENTACION
DRENAJE VENOSO ANOMALO PRESENTACION
 
Nefropatia diabetica
Nefropatia diabetica Nefropatia diabetica
Nefropatia diabetica
 
Urgencias: Sangrado Del Tubo Digestivo Bajo
Urgencias: Sangrado Del Tubo Digestivo BajoUrgencias: Sangrado Del Tubo Digestivo Bajo
Urgencias: Sangrado Del Tubo Digestivo Bajo
 
Absceso hepático amebiano
Absceso hepático amebianoAbsceso hepático amebiano
Absceso hepático amebiano
 
Diarrea crónica. Diagnóstico y tratamiento. 2018
Diarrea crónica. Diagnóstico y tratamiento. 2018Diarrea crónica. Diagnóstico y tratamiento. 2018
Diarrea crónica. Diagnóstico y tratamiento. 2018
 
Coledocolitiasis
Coledocolitiasis Coledocolitiasis
Coledocolitiasis
 
Higado imagenologia
Higado imagenologiaHigado imagenologia
Higado imagenologia
 
Anemia
AnemiaAnemia
Anemia
 
Enfermedades de la aorta
Enfermedades de la aortaEnfermedades de la aorta
Enfermedades de la aorta
 
Colecistitis crónica
Colecistitis crónicaColecistitis crónica
Colecistitis crónica
 
Nefropatía hipertensiva
Nefropatía hipertensivaNefropatía hipertensiva
Nefropatía hipertensiva
 

Destacado (20)

4.GASTROENTEROLOGIA Y CIRUGIA
4.GASTROENTEROLOGIA Y CIRUGIA4.GASTROENTEROLOGIA Y CIRUGIA
4.GASTROENTEROLOGIA Y CIRUGIA
 
2.CARDIOLOGIA
2.CARDIOLOGIA2.CARDIOLOGIA
2.CARDIOLOGIA
 
10.NEUMOLOGIA
10.NEUMOLOGIA10.NEUMOLOGIA
10.NEUMOLOGIA
 
1.HEMATOLOGIA
1.HEMATOLOGIA1.HEMATOLOGIA
1.HEMATOLOGIA
 
8.NEUROLOGIA
8.NEUROLOGIA8.NEUROLOGIA
8.NEUROLOGIA
 
ENARM Obstetricia ginecologia
ENARM Obstetricia ginecologiaENARM Obstetricia ginecologia
ENARM Obstetricia ginecologia
 
Hipertension en el embarazo ACOG 2013
Hipertension en el embarazo ACOG 2013Hipertension en el embarazo ACOG 2013
Hipertension en el embarazo ACOG 2013
 
PEDIATRIA ENARM
PEDIATRIA ENARMPEDIATRIA ENARM
PEDIATRIA ENARM
 
7.DERMATOLOGIA
7.DERMATOLOGIA7.DERMATOLOGIA
7.DERMATOLOGIA
 
6.INFECTOLOGIA
6.INFECTOLOGIA6.INFECTOLOGIA
6.INFECTOLOGIA
 
ENARM COMPENDIO
ENARM  COMPENDIOENARM  COMPENDIO
ENARM COMPENDIO
 
9.ENDOCRINOLOGIA
9.ENDOCRINOLOGIA9.ENDOCRINOLOGIA
9.ENDOCRINOLOGIA
 
3.REUMATOLOGIA
3.REUMATOLOGIA3.REUMATOLOGIA
3.REUMATOLOGIA
 
Planificacion familiar curso enarm cmn siglo xxi 36246001
Planificacion familiar curso enarm cmn siglo xxi 36246001Planificacion familiar curso enarm cmn siglo xxi 36246001
Planificacion familiar curso enarm cmn siglo xxi 36246001
 
Primer simulador enarm 2013 primera parte final
Primer simulador enarm 2013 primera parte finalPrimer simulador enarm 2013 primera parte final
Primer simulador enarm 2013 primera parte final
 
Anatomia esofago enarm
Anatomia esofago enarmAnatomia esofago enarm
Anatomia esofago enarm
 
SEGUNDO SIMULADOR (EJEMPLO)
SEGUNDO SIMULADOR (EJEMPLO)SEGUNDO SIMULADOR (EJEMPLO)
SEGUNDO SIMULADOR (EJEMPLO)
 
Hipospadia; Fimosis; Parafimosis
Hipospadia; Fimosis; ParafimosisHipospadia; Fimosis; Parafimosis
Hipospadia; Fimosis; Parafimosis
 
Balanitis
Balanitis Balanitis
Balanitis
 
Tinnitus
TinnitusTinnitus
Tinnitus
 

Similar a NEFROLOGÍA: Electrolytes y Casos Clínicos

Alteraciones electrolíticas
Alteraciones electrolíticasAlteraciones electrolíticas
Alteraciones electrolíticasAlien
 
(2023-30-11) Manejo de la hiponatremia (ptt).pptx
(2023-30-11) Manejo de la hiponatremia (ptt).pptx(2023-30-11) Manejo de la hiponatremia (ptt).pptx
(2023-30-11) Manejo de la hiponatremia (ptt).pptxUDMAFyC SECTOR ZARAGOZA II
 
Desequilibrio hidroelectrolitico ok
Desequilibrio hidroelectrolitico okDesequilibrio hidroelectrolitico ok
Desequilibrio hidroelectrolitico okeddynoy velasquez
 
HIPOKALEMIA E HIPERKALEMIA presentación (1).pdf
HIPOKALEMIA E HIPERKALEMIA presentación (1).pdfHIPOKALEMIA E HIPERKALEMIA presentación (1).pdf
HIPOKALEMIA E HIPERKALEMIA presentación (1).pdfitzelmoreno97
 
liquidos y electrolitos
liquidos y electrolitos liquidos y electrolitos
liquidos y electrolitos DdCc3
 
Liquidos y electrolitos
Liquidos y electrolitosLiquidos y electrolitos
Liquidos y electrolitosIsabel Rojas
 
LíQuidos Y Electrolitos. Dr. Hernandez
LíQuidos Y Electrolitos. Dr. HernandezLíQuidos Y Electrolitos. Dr. Hernandez
LíQuidos Y Electrolitos. Dr. Hernandezjaime hernandez
 
Lquidos Y Electrolitos
Lquidos Y ElectrolitosLquidos Y Electrolitos
Lquidos Y Electrolitosmelissajj09
 
Alteraciones-Hidroelectrolíticas-en-UCI.pdf
Alteraciones-Hidroelectrolíticas-en-UCI.pdfAlteraciones-Hidroelectrolíticas-en-UCI.pdf
Alteraciones-Hidroelectrolíticas-en-UCI.pdfElizabethClaudi
 
MANEJO DE LIQUIDOS Y ELECTROLITOS.pptx
MANEJO DE LIQUIDOS Y ELECTROLITOS.pptxMANEJO DE LIQUIDOS Y ELECTROLITOS.pptx
MANEJO DE LIQUIDOS Y ELECTROLITOS.pptxssuser72c545
 
Michelle Chavez transtornos liquidos.pdf
Michelle Chavez transtornos liquidos.pdfMichelle Chavez transtornos liquidos.pdf
Michelle Chavez transtornos liquidos.pdfAlexandraChavez41
 
Hipernatremia, definiciones y aspectos a considerar en la práctica clínica
Hipernatremia, definiciones y aspectos a considerar en la práctica clínicaHipernatremia, definiciones y aspectos a considerar en la práctica clínica
Hipernatremia, definiciones y aspectos a considerar en la práctica clínicaAndrés Zúñiga Zapata
 

Similar a NEFROLOGÍA: Electrolytes y Casos Clínicos (20)

HIPONATREMIA DAN.pptx
HIPONATREMIA DAN.pptxHIPONATREMIA DAN.pptx
HIPONATREMIA DAN.pptx
 
Alteraciones electrolíticas
Alteraciones electrolíticasAlteraciones electrolíticas
Alteraciones electrolíticas
 
Liquidos y electrolitos
Liquidos y electrolitos Liquidos y electrolitos
Liquidos y electrolitos
 
(2023-30-11) Manejo de la hiponatremia (ptt).pptx
(2023-30-11) Manejo de la hiponatremia (ptt).pptx(2023-30-11) Manejo de la hiponatremia (ptt).pptx
(2023-30-11) Manejo de la hiponatremia (ptt).pptx
 
Desequilibrio hidroelectrolitico ok
Desequilibrio hidroelectrolitico okDesequilibrio hidroelectrolitico ok
Desequilibrio hidroelectrolitico ok
 
HIPOKALEMIA E HIPERKALEMIA presentación (1).pdf
HIPOKALEMIA E HIPERKALEMIA presentación (1).pdfHIPOKALEMIA E HIPERKALEMIA presentación (1).pdf
HIPOKALEMIA E HIPERKALEMIA presentación (1).pdf
 
liquidos y electrolitos
liquidos y electrolitos liquidos y electrolitos
liquidos y electrolitos
 
Manejo de fluidos y electrolitos
Manejo de fluidos y electrolitosManejo de fluidos y electrolitos
Manejo de fluidos y electrolitos
 
Liquidos y electrolitos
Liquidos y electrolitosLiquidos y electrolitos
Liquidos y electrolitos
 
LíQuidos Y Electrolitos. Dr. Hernandez
LíQuidos Y Electrolitos. Dr. HernandezLíQuidos Y Electrolitos. Dr. Hernandez
LíQuidos Y Electrolitos. Dr. Hernandez
 
Lquidos Y Electrolitos
Lquidos Y ElectrolitosLquidos Y Electrolitos
Lquidos Y Electrolitos
 
Alteraciones-Hidroelectrolíticas-en-UCI.pdf
Alteraciones-Hidroelectrolíticas-en-UCI.pdfAlteraciones-Hidroelectrolíticas-en-UCI.pdf
Alteraciones-Hidroelectrolíticas-en-UCI.pdf
 
Medio interno
Medio internoMedio interno
Medio interno
 
MANEJO DE LIQUIDOS Y ELECTROLITOS.pptx
MANEJO DE LIQUIDOS Y ELECTROLITOS.pptxMANEJO DE LIQUIDOS Y ELECTROLITOS.pptx
MANEJO DE LIQUIDOS Y ELECTROLITOS.pptx
 
Michelle Chavez transtornos liquidos.pdf
Michelle Chavez transtornos liquidos.pdfMichelle Chavez transtornos liquidos.pdf
Michelle Chavez transtornos liquidos.pdf
 
AKI.pptx
AKI.pptxAKI.pptx
AKI.pptx
 
IRA -UNSM-2014
IRA -UNSM-2014IRA -UNSM-2014
IRA -UNSM-2014
 
Anatomia fisiologia
Anatomia fisiologiaAnatomia fisiologia
Anatomia fisiologia
 
28. alteraciones metabolicas de k
28. alteraciones metabolicas de k28. alteraciones metabolicas de k
28. alteraciones metabolicas de k
 
Hipernatremia, definiciones y aspectos a considerar en la práctica clínica
Hipernatremia, definiciones y aspectos a considerar en la práctica clínicaHipernatremia, definiciones y aspectos a considerar en la práctica clínica
Hipernatremia, definiciones y aspectos a considerar en la práctica clínica
 

NEFROLOGÍA: Electrolytes y Casos Clínicos

  • 2.
  • 3.
  • 4.
  • 5. ANION GAP • El Anión GAP es una ECUACIÓN que sirve PARA CALCULAR ANIONES ORGÁNICOS que están presentes a una CONCENTRACIÓN MUY PEQUEÑA como para ser medidas en un ionograma... Partiendo desde el concepto de electro neutralidad, pareciera que los cationes "superan" a los aniones, pero justamente la "brecha" entre ellos esta representados por estos Aniones... La utilidad fisiológica/clínica del Anión Gap, es para orientarte acerca de la POSIBLE ETIOLOGÍA ( causa) DE LA ACIDOSIS METABOLICA ( y SOLO SIRVE PARA ESTO) que el paciente esta cursando... Las AC METAB. pueden ser DE 2 "TIPOS"... por FALTA DE BASE ( el tipo tiene una DIARREA tremenda, esta largando hasta la 1º papilla... ), o por EXCESO DE ACIDO ( esta TOMANDO ASPIRINA como si fueran tic tacs... o CETOACIDOSIS etc.) ( esos son algunos ejemplos... )... Si calculas el AG... SIENDO AG= [NA+]P - ([CL-]P+[HCO3-]P) Los valores NORMALES: AG= 12 +- 2 En caso de que el Anion Gap de AUMENTADO (+ de 14) Entonces estamos viendo una ACIDOSIS METABOLICA POR EXCESO DE ACIDO... Si el AG DA NORMAL, entonces, el paciente tiene un AC METAB. POR FALTA DE BASE... Este tipo de ac metab, también es conocida por ACIDOSIS HIPERCLOREMICA
  • 6.
  • 7.
  • 8. CASO CLINICO A 25-year-old woman presents with nausea and vomiting of 2 days duration. She is not on any medications and was previously well until now. Her physical examination is normal except for a postural drop in her blood pressure from 110/80 mm Hg supine to 90/80 mm Hg standing. Her serum electrolytes are sodium 130 mEq/L, potassium 3 mEq/L, chloride 90 mEq/L, bicarbonate 30 mEq/L, urea 50 mg/dL, and creatinine 0.8 mg/dL. Which of the following electrolytes is most likely to be filtered through the glomerulus but unaffected by tubular secretion? (A) potassium (B) sodium (C) bicarbonate (D) urea (E) creatinine
  • 9. (D) Urea is filtered at the glomerulus, and thereafter, any movement in or out of tubules is a passive process depending on gradients, not secretion. Reabsorption of urea in the distal tubule and collecting duct, when urine flow is reduced, results in the disproportionate elevation of urea nitrogen over creatinine in prerenal azotemia.
  • 10. SODIO • 1. Cuales son las causas de Hiponatremia? • R = Es debida a múltiples patologías como SÍNDROME DE SECRECIÓN INAPROPIADA DE HORMONA ANTIDIURETICA o cirrosis hepática, IC. • 2. Cual es el cuadro clínico de la hiponatremia? • R = Cefalea, calambres, fatiga, letargia, APATIA, ANOREXIA, DESORIENTACION. • 3. Como realizas el diagnostico de origen de hiponatremia? • R = ES en sangre, NA URINARIO < 20 MEQ es debido a perdidas EXTRARENALES y NA URINARIO > 20 MEQ son de causa RENAL • 4. Cual es la etiología de la hipernatremia? • R = Es debida a PERDIDA DE AGUA o retención de sodio. • 5. Cual es el cuadro clínico de la hipernatremia? • R = 1) DEPLECION DE VOLUMEN: Como hipotensión ortostatica, taquicardia, disminución de la turgencia de la piel. 2) VOLUMEN NORMAL: Datos neurológicos como IRRITABILIDAD, CONFUSION , letárgia, HIPERREFLEXIA. • 6. Cual es el manejo de la hipernatremia? • R = DIURÉTICOS y solución glucosada al 5% • 7. Que puede causar hiponatremia isotónica? • R = HIPERLIPIDEMIA, hiperproteinemia • 8. Que causa hiponatremia hipertónica? • R = HIPERGLUCEMIA, manitol, sorbitol, radiocontraste • 9. Que fármaco se utiliza para la hiponatremia que provoca el sx cerebral con perdida de sal? • R = FLUDOCORTISONA
  • 11.
  • 12.
  • 13. SIADH • 10. Que tumoración cancerosa provoca sx de secreción inapropiada de ADH? • R = Carcinoma pulmonar de CÉLULAS PEQUEÑAS en pulmón • 11. Que caracteriza al síndrome de secreción inapropiada de ADH? • R = HIPONATREMIA, osmolaridad <280, OSMOLARIDAD URINARIA >150, función tiroidea y suprarrenal normal, NA URINARIO > 20. • -A 65-year-old woman is admitted to the hospital after a stroke resulting in left hemiparesis. She has no trouble swallowing and is on a full diet. One week later, on routine biochemistry, her sodium is 128 mEq/L and osmolality is 270 mOsm/kg. She has no symptoms and is euvolemic on clinical examination. Urine sodium is 40 mEq/L and urine osmolality is 450 mOsm/kg. • -SIADH is associated with many CNS diseases including meningitis, encephalitis, tumors, trauma, stroke, and acute porphyria. It is assumed that antidiuretic hormone (ADH) in these patients is secreted in response to direct stimulation of the hypothalamic osmoreceptors. • -A 42-year-old man is on amitriptyline for depression. Recently he has been feeling more lethargic and unwell. His clinical examination is normal. Serum sodium is 125 mEq/L and osmolality is 260 mOsm/kg, and urine sodium is 40 mEq/L and osmolality is 450 mOsm/kg: • -Amitriptyline is one of the psychoactive drugs that cause SIADH. Others include phenothiazines, serotonin reuptake inhibitors, and monoamine oxidase inhibitors (MAOIs). Antineoplastic drugs such as vincristine and cyclophosphamide also cause SIADH, as does the hypoglycemic agent chlorpropamide • 12. Que fármaco inhibe la acción de la ADH? • R = DEMECLOCICLINA que actúa sobre receptor V2 de la ADH.
  • 14. CASO CLINICO A 77-year-old man with a mass in the lung develops asymptomatic hyponatremia. His JVP is 4 cm, heart sounds are normal, and the lungs are clear. The urine sodium is 64 mEq/L and osmolality 550 mOsm/kg. Which of the following is the most likely diagnosis? (A) nephrotic syndrome (B) syndrome of inappropriate antidiuretic hormone (SIADH) production (C) renal metastases from lung cancer (D) lung metastases from hypernephroma (E) renal tubular acidosis (RTA)
  • 15. (B) The urine osmolality in patients with SIADH need not be hypertonic to plasma, but only inappropriately high compared with serum. The major characteristics of SIADH include hyponatremia, volume expansion without edema, natriuresis, hypouricemia, and normal or reduced serum creatinine level, with normal thyroid and adrenal function. (Kasper, pp. 2102–2103)
  • 16. POTASIO• 13. Cuales son las causas de la hipokalemia? • R = Incremento en el recambio celular, disminución de la ingesta o un aumento de su excreción • 14. Que medida de laboratorio es útil para el diagnostico de hipokalemia de origen renal o extrarenal? • R = EL GRADIENTE TRANSTUBULAR DE POTASIO (TTKG), el cual si es < 2 TRADUCE HIPOKALEMIA NORENAL y UNO > 10 ES DEBIDO A PERDIDAS RENALES. EL K URINARIO <25 MEQ/DIA es NO RENAL y K URINARIO > 30 MEQ/DIA traduce perdida de ORIGEN RENAL. • 15. Cuales son lo síntomas de hipokalemia? • R = DEBILIDAD MUSCULAR, fatiga, CALAMBRES. <2.5 Meq parálisis flácida, HIPORREFLEXIA, hipercapnia y rabdomiolisis. • 16. Que elementos participan en la captación de potasio? • R = La insulina en presencia de glucosa, estimulación B adrenérgica • 17. Cual es el regulador de potasio mas importante? • R = ALDOSTERONA facilita la excreción urinaria de K en los TCD. • 18. Cuales son las causas de la hiperkalemia? • R = > 5.5 mEq. DISMINUCION DE LA EXCRESION: HIPOALDOSTERONISMO, IRA o IRC, ESPIRONOLACTONA, IECA, B bloqueador. Desplazamiento de K al interior: Rabdomiolisis, hemolisis y ACIDOSIS METABOLICA • 19. Cual es el cuadro clínico de hiperkalemia y EKG? • R = DEBILIDAD MUSCULAR, distención abdominal, DIARREA. EKG con onda T picuda, QT corto y depresión del segmentoT. • 20. Cual es el manejo de la hiperkalemia? 1) INSULINA + GLUCOSA AL 10%, SALBUTAMOL inhalado, GLUCONATO DE CALCIO IV, HCO3 que produce ingreso a la celula de K. 2) EL TRATAMIENTO AGUDO CON GLUCONATO DE CALCIO DEBE RESERVARSEcuando se presentan manifestaciones cardiacas de toxicidad celular por exceso de potasio (calcitosis).
  • 17.
  • 18.
  • 19.
  • 20.
  • 22. CASO CLINICO A 60-year-old woman with heart failure and normal renal function is started on furosemide (Lasix) 80 mg/day. She notices a good diuretic response every time she takes the medication. Afew weeks later, she is feeling unwell because of fatigue and muscle weakness, but her heart failure symptoms are better. Which of the following is the most likely explanation for her muscle weakness? (A) hyponatremia (B) hypernatremia (C) hypokalemia (D) hyperkalemia (E) anemia
  • 23. (C) Hypokalemia can result in paralytic ileus, rhabdomyolysis, weakness, and cardiac repolarization abnormalities. It is a common complication along with hyponatremia of starting patients on diuretics. (Kasper, p. 260)
  • 24. CALCIO • 21. Cuales son las 2 principales causas de hipocalcemia? • R = ACCIÓN INSUFICIENTE DE PTH O VIT D siendo la mas frecuente IRA o IRC • 22. Cuales son los signos y síntomas de HIPOCALCEMIA? • R = Calambres, TETANIA, laringoespasmo, convulsiones, SIGNO DE CHEVOSTEK Y TROUSSEAU POSITIVO. EKG con PROLONGACIÓN DEL ST • 23. Cual es el manejo de la hipocalcemia? • R = En presencia de TETANIA, ARRITMIAS o CONVULCIONES se indica GLUCONATO DE CALCIO en infusión. ASINTOMÁTICOS se da CALCIO Y VIT D • 24. Cuales son las causas mas comunes de HIPERCALCEMIA? • R = HIPERPARATIROIDISMO primario y provenientes de malignidad en 90%. Fármacos, IR, trasplante renal, SINDROME DE LECHE-ALCALI (caracterizado por hipercalcemia, alcalosis, nefrocalcinosis e IR CAUSADO POR CONSUMO EXCESIVO DE CA Y ANTIÁCIDOS). • 25. Cuales son los síntomas de hipercalciemia? • R = Comienza >10.4 mg/dl, ESTREÑIMIENTO, poliuria, nausea, vomito, ULCERA PÉPTICA, debilidad, depresión, LETARGO. EXTRASÍSTOLES VENTRICULARES. EKG con intervalo QT ACORTADO • 26. Cual es el manejo de hipercalcemia? • R = La hipercalcemia tiende a depletar el volumen por DM insípida nefrogenica, por lo tanto se debe restablecer EUVOLEMIA + FUROSEMIDE. Hipercalcemia por MALIGNIDAD SE USA BIFOSFONATOS, DIÁLISIS. Clorhidrato de Cinacalcet que suprime la secreción de PTH y disminuye calcio.
  • 25. FOSFATO • 27. Que situaciones disminuyen la absorción del TCP de fosfato? • R = Expansión de volumen, glucocorticoides, DISFUNCIÓN TUBULAR PROXIMAL COMO EN EL SX DE FANCONI. La hormona del crecimiento aumenta su reabsorción. • 28. Cuales son las causas de hipofosfatemia? • R = Fármacos fosfaturicos: XANTINAS, TEOFILINA, esteroides. HIPERPARATIROIDISMO E HIPERGLUCEMIA. Alcalosis respiratoria, ADMINISTRACIÓN DE INSULINA Y GLUCOSA SIMULTÁNEOS y SÍNDROME DE HUESO HAMBRIENTO (posterior a para tiroidectomía para corrección de hiperparatiroidismo, el deposito masivo de fosforo y calcio dan como resultado hipocalcemia y fosfatemia) • 29. A que se debe la debilidad muscular o rabdomiolisis en hipofosfatemia? • R = Aumento de la afinidad de la Hb al O2 por difosfoglicerato en el eritrocito, por lo tanto, disminuye la oxigenación tisular y metabolismo celular. • 30. Cual es el manejo de la hipofosfatemia? • R = Si se administra FOSFATO RÁPIDAMENTE puede OCASIONAR HIPERCALCEMIA y SE PREFIERE VO. • 31. Cuales son las causas más comunes de hiperfosfatemia? • R = HIPOPARATIROIDISMO, IR, excreción renal de fosforo disminuida, aumento de fosforo al espacio EC y administración de sales de fosforo o Vitamina D. • 32. Cual es el cuadro clínico de la hiperfosfatemia? • R = Esta RELACIONADO A LA HIPERCALCEMIA • 33. Cual es el manejo de la hiperfosfatemia? • R = RESTRICCIÓN de fosforo en dieta, QUELANTES de fosforo en IR como SEVELAMER O ALOGLUTAMOL.
  • 26. MAGNESIO • 34. Cuales son las causas de hipomagnesemia? • R = Aumento del volumen intravascular, DIURÉTICO, cisplatino, aminoglucósido y anfotericina B. • 35. Cuales son los signos y síntomas de hipomagnesemia? • R = DEBILIDAD, CALAMBRES, temblor, movimiento atetoide, arritmias ventriculares, confusión. • 36. Cuales son los datos de lab que te indican hipomagnesemia? • R = Excreción urinaria es mas confiable, >10-30 mg/dl. La HIPOMAGNESEMIA SIEMPRE SE ASOCIA A HIPOCALCEMIA E HIPOKALEMIA. EKG intervalo QT PROLONGADO. Supresión de la PTH. • 37. Cual es el manejo de la hipomagnesemia? • R = Mg IV o IM. OXIDO DE Mg VO. • 38. Cuales son las causas de hipermagnesemia? • R = IR o ingesta de Mg • 39. Cuales son los síntomas de hipermagnesemia? • R = Debilidad muscular, DISMINUCIÓN DE ROTS y confusión. RUBOR FACIAL, bradiarritmias. • 40. Cuales son las manifestaciones EKG de hipermagnesemia? • R = Aumento del INTERVALO PR, ENSANCHAMIENTO DEL QRS y ondas T ACUMINADAS. • 41. Cual es el manejo de la hipermagnesemia? • R = Ca en IRC, el cual actúa como antagonista
  • 27. BENCE JONES • 42. Que cifras de urianalisis en 24 hrs revela proteinuria? • R = >150 mg/24 hrs • 43. Que enfermedad se relaciona con la proteinuria de Bence Jones? • R = Mieloma múltiple • -A 67-year-old man presents with symptoms of renal colic. Plain x-rays of the abdomen reveal no obvious stone. An intravenous pyelogram (IVP) is ordered to confirm the clinical diagnosis. Which of the following coexisting medical conditions increases the risk of contrast-induced nephropathy? • (A) hyperparathyroidism • (B) pyelonephritis • (C) nephrolithiasis • (D) hypernephroma • (E) multiple mieloma • -The danger of ARF after IVP has led to caution, especially in patients with multiple myeloma. The patient should not be dehydrated if IVP is necessary. The risk is also increased in patients with diabetes mellitus or chronic renal failure. • -A74-year-old man presents with fatigue, shortness of breath on exertion, and back and rib pain, which is made worse with movement. Investigations reveal he is anemic, calcium, urea, and creatinine are elevated. X-rays reveal multiple lytic lesions in the long bones and ribs, and protein electrophoresis is positive for an immunoglobulin G (IgG) paraprotein. Which of the following is the most likely mechanism for the renal injury? • (A) plasma cell infiltrates • (B) tubular damage by light chains • (C) glomerular injury • (D) vascular injury by light chains • (E) uric acid crystals • -In multiple myeloma, tubular damage by light chains is almost always present. The injury is a direct toxic effect of the light chains or indirectly from the inflammatory response. Infiltration by plasma cells and glomerular injury is rare. Hypercalcemia may produce transient or irreversible renal damage as do amyloid and myeloma cell infiltrates.
  • 28. IRA• 44. Cual es la causa mas común de IRA? • R = Azoemia prerrenal por HIPO PERFUSIÓN • 45. A que se debe el daño renal ocasionado por la toma persistente de AINES? • R = Hay aumento en la proporción de oxido nítrico, BLOQUEO DE LA PRODUCCIÓN DE PROSTAGLANDINAS, inhibición de los leucotrienos, producción elevada de glucoproteina IIb/IIIa y disminución en la producción de renina. • 46. Como se encuentra en la AZOEMIA PRERRENAL la relación BUN/ Creatinina? • R = > 20 • 47. Como se encuentra el FENA en la IRA? • R = Prerenal <1% y postrenal > 2%. • Cual de las siguientes patologías constituyen indicacion de dialisis precoz en IRA? • R = Hiperkalemia refractaria al tratamiento, sobrecarga de volumen no manejable con tratamiento medico, ACIDOSIS METABOLICA INCONTROLABLE y pericarditis urémica. La IRA por contraste NO es una indicación. • Cuales hallazgoz analiticos son comunes en IRA? • Hiperkalemia, acidosis metabólica, hiperfosfatemia e hipermagnesemia • -A 46-year-old woman with nausea and vomiting presents to hospital because of lightheadedness when standing and decreased urine output. She looks unwell; the blood pressure supine is 90/60 mm Hg and 80/60 mm Hg when standing. Her abdominal, heart, and lung examinations are normal. Which of the following laboratory values suggests prerenal azotemia in this patient? • (A) markedly elevated urea, unchangedvcreatinine • (B) unchanged urea, elevated creatinine • (C) little change in either creatinine or ureavfor several days after oliguria develops • (D) urea/creatinine ratio of 10 • (E) urea/creatinine ratio of 25 • -The ratio of BUN/creatinine is usually <10–15 in intrinsic renal disease and >20 in prerenal azotemia. • -A74-year-old man from a nursing home is not feeling well and is confused. He is not able to give any reliable history. His serum sodium is 120 mEq/L and osmolality is 265 mOsm/kg. • -The combination of ECF volume contraction with high urinary sodium (20 mmol/L) suggests renal fluid loss. This is commonly caused by diuretics or glucosuria.
  • 29.
  • 30.
  • 31.
  • 32.
  • 33.
  • 34. CASO CLINICO A 67-year-old man presents with symptoms of renal colic. Plain x-rays of the abdomen reveal no obvious stone. An intravenous pyelogram (IVP) is ordered to confirm the clinical diagnosis. Which of the following coexisting medical conditions increases the risk of contrast-induced nephropathy? (A) hyperparathyroidism (B) pyelonephritis (C) nephrolithiasis (D) hypernephroma (E) multiple myeloma
  • 35. (E) The danger of ARF after IVP has led to caution, especially in patients with multiple myeloma. The patient should not be dehydrated if IVP is necessary. The risk is also increased in patients with diabetes mellitus or chronic renal failure. (Kasper, pp. 1646–1647)
  • 37. NTA • 48. A que padecimiento se le conoce como NTA? • R = IRA por lesiones tubulares siendo las principales causas la isquémica y por toxinas • 49. Que nefrotoxinas endógenas ocasionan NTA? • R = Productos contenedores de HEM, ACIDO ÚRICO, MIOGLOBINURIA por RABDOMIOLISIS • 50. Cuales son los datos de lab de NTA? • R = ORINA COLOR CAFÉ LODOSO por Hb y cilindros epiteliales de túbulos renales • 51. Cual es el manejo de la NTA? • R = Se debe prevenir hipercalcemia, FUROSEMIDE, RESTRICCIÓN DE PROTEÍNAS DIETÉTICAS Y DIÁLISIS EN PACIENTE CRITICO • Cual es la complicación mas común y mas temida de la NTA? • R = Falla cardiaca
  • 38. CASO CLINICO A 74-year-old woman develops acute sepsis from pneumonia and is admitted to the intensive care unit because of hypotension. She is started on antibiotics, and her blood pressure is supported with intravenous normal saline. Despite this she remains oliguric and develops ARF. Her urinalysis has heme-granular casts and the urine sodium is 56 mEq/L. Which of the following is the most likely cause of her ARF? (A) nephrotoxic antibiotics (B) acute infectious GN (C) acute tubular necrosis (ATN) (D) contrast nephropathy (E) cholesterol emboli
  • 39. (C) ATN is a common complication of prolonged hypotension and ischemic injury to the renal tubules. Heme-granular “muddy brown” casts are consistent with ATN. Peritoneal dialysis is preferred with cerebral trauma as well as with severe heart failure because of risk of hemorrhage or hypotension with hemodialysis. In particular, peritoneal dialysis does not require any anticoagulation and is safer if head trauma has occurred. (Kasper, pp. 1645–1646)
  • 40. NEFRITIS INSTERSTICIAL AGUDA • 52. Que es la nefritis intersticial aguda? • R = Respuesta inflamatoria intersticial con edema y posible daño celular • 53. Cuales son las causas de nefritis intersticial? • R = FÁRMACOS 70%. Infecciosas e inmunitarias • 54. Cuales son los datos clínicos de nefritis intersticial aguda? • R = Se caracteriza por un DETERIORO ABRUPTO DE LA FUNCIÓN RENAL acompañado de poliuria, Nicturia, ACIDOSIS METABOLICA y glucosuria. FIEBRE90%, ARTRALGIAS80%, EXANTEMA20%. Bh: EOSINOFILIA EN FASE AGUDA. EGO con EOSINOFILURIA, hematuria, leucocituria y cilindros leucocitarios. • 55. Cuales son los hallazgos en la biopsia con nefritis intersticial aguda? • R = Infiltración de células inflamatorias en intersticio renal con edema, predominan mononucleares y LT, GRANULOMAS NO CASEIFICANTES. • 56. Cual es el manejo de la nefritis intersticial aguda? • R = RETIRAR EL FACTOR predisponente. Puede llegar a REQUERIR DIÁLISIS, MEDIDAS DE SOSTEN, METILPREDNISOLONA en NI POR FÁRMACOS • 57. Que caracteriza a la necrosis intersticial crónica? 1. Ausencia de proteinuria e hipoalbuminemia 2. Piuria estéril y leucocitosis mas que hematuria 3. Poliuria y Nicturia 4. Otros defectos tubulares como ATR y osteomalacia.
  • 41. GLOMERULONEFRITIS • 58. Cuales son las causas de glomerulonefritis? • R = Nefropatía por IgA (enf. De BERGUER), autoinmunitaria o infecciosa. • 59. Cuales son los datos clínicos de glomerulonefritis? • R = HIPERTENSIÓN, EDEMA periorbitario o escrotal y PROTEINURIA. • 60. Cuales son los hallazgos de lab en GN? • R = Hematuria, PROTEINURIA, cilindros leuco-eritrocitarios. Complemento C3,C4 CH50. Ac vs MBG, ANCA • 61. Cual es el manejo de la GN? • R = Prednisona
  • 42.
  • 43.
  • 44.
  • 45.
  • 46. IRC • 62. Cual es la clasificación de la IRC? • ESTADIO FG • I 90 • II 60 – 89 • III 30 – 59 • IV < 15 O DIALISIS • 63. Cual es el manejo restrictivo de IRC? • R = Restricción de proteínas, restricción de Na y H2 O, restricción de Mg entre otros. • Que pasa con la capacidad de dilución y de concentración en la IRC? • La capacidad de DILUCIÓN se deteriora antes que la de CONCENTRACIÓN.
  • 47.
  • 48.
  • 49.
  • 50. SX NEFRITICO Y NEFROTICO • 64. Que caracteriza al síndrome nefrítico? • R = Proteinuria < 3.5 GR/ DÍA, Daño glomerular que ocasiona HEMATURIA, edemas peri orbitarios, escrotal e HIPERTENSION. • 65. Cuales son los datos de lab del sx nefrítico? • R = HEMATURIA con eritrocitos dismorficos y CILINDROS HEMATICOS, se solicitan ANCA, Ac vs MBG Y C3. • 66. Cual es el manejo del síndrome nefrítico? • R = DISMINUIR LA TA, corregir causa de fondo, restricción de H20- diuréticos- diálisis, METILPREDNISOLONA.
  • 51. • 67. Que entidades patológicas producen sx nefrotico? • R = DM, amiloidosis y LES. Presentándose como GLOMERULONEFRITIS MEMBRANOSA EN ADULTOS y ENFERMEDAD DE CAMBIOS MÍNIMOS EN NIÑOS. • -A 64-year-old man presents with weight gain, shortness of breath, easy bruising, and leg swelling. On examination, his blood pressure is 140/80 mm Hg, pulse 100/min, JVP 4 cm, heart sounds normal, and lungs are clear. There is a 3+ pedal and some periorbital edema. Investigations include a normal chest x-ray (CXR), electrocardiogram (ECG) with low voltages, anemia, high urea and creatinine, and 4 g/day of protein in the urine. A renal biopsy, which shows nodular deposits that have an apple-green birefringence under polarized light when stained with Congo red. Which of the following is the most likely diagnosis? • (A) amyloidosis • (B) multiple myeloma • (C) diabetic nephropathy • (D) minimal change disease • (E) immunoglobulin A (IgA) nephropathy • - Renal amyloidosis can be primary (AL) or secondary amyloidosis (AA). The hallmark finding, nephrotic syndrome, is present in 25% of patients at presentation and probably develops ultimately in over 50%. The apple-green birefringence deposits under polarized light are diagnostic of amyloidosis, and not seen in any other renal disease. • -A 42-year-old man notices leg and facial swelling but no other symptoms. His examination is pertinent for 3+ pedal edema including periorbital edema. A 24-hour urine collection reveals 5 g of proteinuria. Which of the following is the most likely diagnosis? • (A) sickle cell disease • (B) medullary sponge kidney • (C) radiation nephritis • (D) staphylococcal infection • (E) amyloid disease • -In addition to amyloid disease, other conditions associated with nephrotic syndrome are secondary syphilis, malaria, and treatment with gold salts. Minimal change nephrotic syndrome, focal glomerular sclerosis, membranous nephropathy, and membranoproliferative GN are the primary renal diseases that present as ephritic syndrome.
  • 52. • 68. Cuales son los signos y síntomas del síndrome nefrótico? • R = EDEMA periférico por albumina > 3 GR/DL que puede llegar a anasarca. • 69. Cuales son los datos de lab del síndrome nefrótico? • R = EGO: en racimos de uvas por LIPIDURIA, PROTEINURIA >3.5 gr/dl/ 24 hrs. QS: ALBUMINA <3 GR/ DL, proteínas totales > 6 gr/dl , HIPERLIPIDEMIA. • 70. Cual es el manejo del síndrome nefrótico? • R = PREDNISONA, restricción de Na para controlar edema, diuréticos. Hay riesgo de IRC con proteinuria > 5 g/24 hrs. • 71. De que manera afecta la coagulación el sx nefrótico? • R = ESTADO HIPERCOAGULABLE cuando la ALBUMINA SE ENCUENTRA <2 GR/DL, se requiere de anti coagulación
  • 53.
  • 54.
  • 55.
  • 56.
  • 57.
  • 58. CASO CLINICO A 64-year-old man presents with weight gain, shortness of breath, easy bruising, and leg swelling. On examination, his blood pressure is 140/80 mm Hg, pulse 100/min, JVP 4 cm, heart sounds normal, and lungs are clear. There is a 3+ pedal and some periorbital edema. Investigations include a normal chest x-ray (CXR), electrocardiogram (ECG) with low voltages, anemia, high urea and creatinine, and 4 g/day of protein in the urine. Arenal biopsy, which shows nodular deposits that have an apple-green birefringence under polarized light when stained with Congo red. Which of the following is the most likely diagnosis? (A) amyloidosis (B) multiple myeloma (C) diabetic nephropathy (D) minimal change disease (E) immunoglobulin A (IgA) nephropathy
  • 59. (A) Renal amyloidosis can be primary (AL) or secondary amyloidosis (AA). The hallmark finding, nephrotic syndrome, is present in 25% of patients at presentation and probably develops ultimately in over 50%. The apple-green birefringence deposits under polarized light are diagnostic of amyloidosis, and not seen in any other renal disease. (Kasper, p. 1689)
  • 60. CASO CLINICO A 42-year-old man notices leg and facial swelling but no other symptoms. His examination is pertinent for 3+ pedal edema including periorbital edema. A24-hour urine collection reveals 5 g of proteinuria. Which of the following is the most likely diagnosis? (A) sickle cell disease (B) medullary sponge kidney (C) radiation nephritis (D) staphylococcal infection (E) amyloid disease
  • 61. (E) In addition to amyloid disease, other conditions associated with nephrotic syndrome are secondary syphilis, malaria, and treatment with gold salts. Minimal change nephrotic syndrome, focal glomerular sclerosis, membranous nephropathy, and membranoproliferative GN are the primary renal diseases that present as nephrotic syndrome. (Kasper, p. 1689)
  • 62. GLOMERULONEFRITIS RP Y PI • 72. Que distingue a la GN rápidamente progresiva? • R = Es una situación clínica en la cual el daño glomerular es tan agudo que la función renal se deteriora en días o semanas de manera rápida y progresiva y también cuando >50% DE LOS GLOMÉRULOS contienen FORMAS SEMILUNARES en la BIOPSIA. • 73. Cual es el manejo de la GN rápidamente progresiva? • R = METILPREDNISOLONA sola combinada con CICLOFOSFAMIDA • 74. Que enfermedades mas comúnmente preceden a la GN infecciosa? • R = IMPÉTIGO y FARINGITIS. • 75. Cual es el cuadro clínico de la GNPI? • R = Oliguria, edema e hipertensión
  • 63.
  • 64. CASO CLINICO A 24-year-old woman presents with nausea, vomiting, anorexia, and gross hematuria. She had a sore throat 2 weeks ago that resolved on its own. On examination, her blood pressure is 160/90 mm Hg, pulse 90/min, JVP is 7 cm, heart sounds are normal, there is 1+ pedal edema, and the lungs are clear. She has a renal biopsy. Which of the following electron microscopy findings on the renal biopsy is most likely in keeping with poststreptococcal GN? (A) diffuse mesangial deposits (B) no deposits (C) electron-dense endothelial deposits (D) closed capillary lumen (E) subepithelial humps
  • 65. (E) These humps are discrete, electron-dense nodules that persist for about 8 weeks and are highly characteristic of the disease. Light microscopy reveals diffuse proliferation, and immunofluorescence reveals granular immunoglobulin G (IgG) and C3. Most patients will recover spontaneously. (Kasper, pp. 1680–1681
  • 66. CASO CLINICO A 19-year-old man presents with malaise, nausea, and decreased urine output. He was previously well, and his physical examination is normal except for an elevated jugular venous pressure (JVP) and a pericardial rub. His electrolytes reveal acute renal failure (ARF). Which of the following findings on the urinalysis is most likely in keeping with acute glomerulonephritis (GN)? (A) proteinuria (B) white blood cell casts (C) granular casts (D) erythrocyte casts (E) hyaline casts
  • 67. (D) Both granular and erythrocyte casts are present, but the latter indicate bleeding from the glomerulus and are most characteristically seen. Red cells reach the urine probably via capillary wall “gaps” and form casts as they become embedded in concentrated tubular fluid with high protein content. Proteinuria is invariably present but is not as specific. (Kasper, pp. 250–251)
  • 68. CRIOGLOBULINEMIA • 76. Que caracteriza a la crioglobulinemia y con que enfermedades se relaciona? • R = Es una vasculitis de pequeños vasos DEBIDO A INMUNOGLOBULINAS que se PRECIPITAN CON EL FRIO relacionándose mucho con el VHC +++ mas que con el VHB, endocarditis o LES. • 77. Cual es el cuadro clínico de la crioglobulinemia? • R = PURPURA 90 %, ARTRITIS 80% , Neutropenia 70 %, DANO RENAL 50% (de este el 80% se manifiesta como GLOMERULONEFRITIS MEMBRANOPROLIFERATIVA Y TROMBOS EN EL INTERIOR DE LOS CAPILARES GLOMERULARES) • 78. Cual es el manejo de la crioglobulinemia? 1) Proteinuria y daño renal leve dándose TRATAMIENTO SINTOMÁTICO Y ESPECIFICO 2) PARA VHC SI ES QUE ESTA ASOCIADO con origen se da INTERFERON Y RIVABIRINA, 3) Si la PROTEINURIA se encuentra en RANGO NEFRÓTICO se da METILPREDNISOLONA, PLASMAFERESIS O RITUXIMAB.
  • 69. NEFROPATÍA POR IgA O SX DE BERGUER • 79. Que caracteriza a la nefropatía por IgA o sx de Berguer? • R = Se deposita IgA en el mesangio glomerular, observándose la MISMA LESIÓN EN PURPURA DE HENOCH- SCHONLEIN • 80. Cuales son los signos de nefropatía por IgA? • R = HEMATURIA “SINFARINGITICA” con orina color COCA-COLA 100%. IVRS50%, GI 10% • 81. Que datos se encuentran en la biopsia en la nefropatía por IgA ? • R = DEPOSITOS DE IgA acompañados de C3 E IgG. • 82. Cual es el manejo de la nefropatía por IgA? 1) IECAS, ESTEROIDES. 2) En caso de proteinuria se combina CICLOFOSFAMIDA CON AZATRIOPINA. • 83. Cual es el pronóstico de nefropatía por IgA? • R = BUENO, pero es MALO para los que desarrollan SEMILUNAS EN LA BIOPSIA
  • 70. GN PAUCINMUNITARIA • 84. En que enfermedad se presenta GN paucinmunitaria? 1) GRANULOMATOSIS DE WEGENER, 2) ENFERMEDAD DE CHURG- STRAUSS, 3) POLIANGEITIS MICROCITICA. • 85. Que patogénesis se encuentra hubicada en 80% de las glomerulonefritis paucinmunitarias? • R = Asociada a ANCA • 86. Cuales son los datos de laboratorio de la GN paucinmunitaria? • R = Patron citoplasmático c-ANCA, patrón perinuclear p-ANCA. • 87. Cual es el manejo de la GN paucinmunitaria? • R = ESTEROIDES, CICLOFOSFAMIDA.
  • 71. AFECCION RENAL X ENF SISTEMICA
  • 72.
  • 73.
  • 76. ENFERMEDAD DE CAMBIOS MÍNIMOS • 88. Cual es la etiología de la enfermedad de cambios mínimos? • R = IDIOPÁTICO. MAS FRECUENTE EN NIÑOS. Hay una teoría que implica desorden de los linfocitos T. Se ha encontrado asociación con procesos virales/ parasitarios y uso de AINES, amitriptilina y RIFAMPICINA. • -A 33-year-old man from Southeast Asia, without HIV infection, is diagnosed as having pulmonary tuberculosis. He is started on multiple medications, including INH and rifampin. Three months later, he has developed edema. Liver tests are normal, and serum creatinine is increased by 30% over baseline. Urinalysis reveals 4+ proteinuria. A renal biopsy is performed: • -Rifampin can cause minimal change disease as well as more severe renal damage. The described case is typical for minimal change GN with nephrotic syndrome. Drug-induced minimal change GN frequently has an associated interstitial nephritis. INH is not usually associated with renal disease. • 89. Cuales son los signos y síntomas de la enfermedad de cambios mínimos? • R = Manifestaciones del SÍNDROME NEFRÓTICO • 90. Que datos histológicos presenta la enfermedad de cambios mínimos? • R = A la microscopia NO REVELA LESIONES GLOMERULARES o solo una mínima prominencia mesangial segmentaria y focal. • 91. Cual es el tratamiento de elección en la enfermedad de cambios mínimos? • R = ESTEROIDES, CICLOFOSFAMIDA en caso de responder a esteroides hasta completar 12 SEMANAS
  • 77.
  • 78. NEFROPATÍA MEMBRANOSA • 92. Que es la nefropatía membranosa? • R = Enfermedad glomerular por DEPOSITO DE IgG y complemento constituyendo la forma mas COMÚN DE SÍNDROME NEFRÓTICO EN ADULTOS, es idiopático 70% y el resto se debe a virus, neoplasias, drogas o enfermedades autoinmunes • 93. Cual es el cuadro clínico de la nefropatía membranosa? • R = SÍNDROME NEFRÓTICO. • 94. Que etiología tiene la NM secundaria? • R = LES, VHC Y ENDOCARDITIS son las mas comunes. Hepatitis B, sífilis, cáncer, penicilamina y captopril. • 95. Cuales son los signos y síntomas de NM secundaria? • R = Relacionados a SX NEFRÓTICO • 96. Que revela la biopsia e inmunohistoquimica en la GMN? • R = HISTOLOGIA: PROLIFERACIÓN EN FORMA DE PARCHES, infiltrado leucocitario intraglomerular y necrosis intracapilar. INMUNOHISTOQUIMICA: IgG siempre presente, IgM e IgA. • 97. Cual es el manejo de la nefropatía membranosa? • R = PROTEINURIA < 4 GR DAS IECAS CON META DE TA 125/75. PROTEINURIA > 4 GR y < 8 GR SIN DETERIORO DE LA FUNCIÓN RENAL EL TRATAMIENTO SE ENCAMINA A DISMINUIR LA PROTEINURIA CON IECAS Y OBSERVAR POR 6M.
  • 79.
  • 80. GLOMERULONEFRITIS FOCAL Y SEGMENTARIA • 98. Cual es la etiología de la Glomerulonefritis focal y segmentaria? • R = Primaria (idiopática) o secundaria debido a otras patologías como agenesia renal, ENFERMEDAD DE CÉLULAS FALCIFORMES o por VIH. • 99. Dentro de las diferentes variedades de Glomerulonefritis focal y segmentaria cuales producen mas proteinuria? • R = LA VARIEDAD CELULAR Y COLAPSANTE en el 80% de los casos de síndrome nefrótico • 100. Cual es el cuadro clínico de la Glomerulonefritis focal y segmentaria? • R = PROTEINURIA asintomática o síndrome nefrótico completo • 101. Cuales son los datos de laboratorio y biopsia de la Glomerulonefritis focal y segmentaria? • R = La PROTEINURIA puede oscilar DESDE <1 GR HASTA 20-30 GR y en la biopsia los hallazgos de HISTOLOGÍA COMO SON LA VARIEDAD CELULAR Y COLAPSANTE. • 102. Cual es el manejo de la Glomerulonefritis focal y segmentaria? 1) LOS QUE TENGAN PROTEINURIA SUBNEFROTICA 2-3 GR/ no se les dará tratamiento inmunosupresor, siendo el principal tratamiento el control de la TA CON META DE 130/80, UTILIZANDO IECAS, hipolipemiantes, DIURÉTICOS EN CASO DE EDEMA. 2) En los pacientes que se presentan con SÍNDROME NEFRÓTICO SE USA PREDNISONA, CONTROL DE LA TA CON META DE 125/75, ANTICOAGULANTES (>10 GR/DIA) y en caso de RESISTENCIA SE USARA CICLOSPORINA. • 103. Cual es el pronostico de la Glomerulonefritis focal y segmentaria? • R = MALO SI LA PROTEINURIA ES > 10 GR/DIA
  • 81. ENFERMEDAD DE LIDDLE • 104. Que es la enfermedad de Liddle? • R = AD. El síndrome de Liddle (o SEUDOHIPERALDOSTERONISMO) es un trastorno AD caracterizado por HIPERTENSIÓN SENSIBLE A SAL con expansión de volumen, HIPOKALEMIA y ALCALOSIS METABOLICA, con ACTIVIDAD DE RENINA Y ALDOSTERONA EXTREMADAMENTE BAJAS y FG NORMAL. • 105. Cual es el cuadro clínico del síndrome de Liddle? • R = Cansancio, poliuria, polidipsia y cefalea. • 106. Cual es el manejo del síndrome de Liddle? • R = TRIAMTERENO y RESTRICCIÓN NA CON SUPLEMENTOS DE K
  • 82. SÍNDROME DE GITELMAN • 107. Que es el síndrome de Gitelman? • R = Se caracteriza por HIPOKALEMIA e HIPOMAGNESEMIA, además de contar con HIPOCALCIURIA, perdida de sal y ALCALOSIS METABÓLICA con HIPERALDOSTERONISMO, HIPERRENINEMIA, asi como HIPERTROFIA E HIPERPLASIA DEL APARATO YUXTAGLOMERULAR. • 108. Cual es el cuadro clínico del síndrome de Gitelman? • R = Debilidad muscular y DERMATITISinespecífica. ESPASMOS DISTALES ocurren EN PERIODOS DE FIEBRE, vomito o diarrea. Algunos sufren condrocalcinosis. • 109. Cual es el principal diagnostico diferencial del síndrome de Gitelman? • R = SINDROME DE BARTTER, donde la RESPUESTA A DIURÉTICO DE ASA es obviamente preservada en el SÍNDROME DE GITELMAN. • 110. Cual es el manejo del síndrome de Gitelman? • R = SUPLEMENTO DE POTASIO Y MAGNESIO, además de TRIAMTERENO, amilorida o espironolactona.
  • 83. SÍNDROME DE BARTTER • 111. Que es el síndrome de Bartter? • R = AR. Se caracteriza por HIPOKALEMIA GRAVE, perdida de saly ALCALOSIS METABÓLICAcon HIPERALDOSTERONISMO, HIPERRENINEMIA, así como HIPERTROFIA E HIPERPLASIA DEL APARATO YUXTAGLOMERULAR. Casi TODOS TIENEN NEFROCALCINOSIS por la hipercalciuria. • 112. Cual es el cuadro clínico del síndrome de Bartter? • R = Tipo I y II: Las manifestaciones tempranas incluyen POLIHIDRAMNIOS, retardo en el crecimiento, polidipsia, deshidratación, avidez por sal y debilidad muscular. En el tipo III: Se manifiesta en la INFANCIA similar al Sx de Gitelman. El tipo IV: TODO LO ANTERIOR MAS SORDERA SENSORINEURAL E IRC TEMPRANA. • 113. Cual es el manejo del síndrome de Bartter? • R = Grandes cantidades de POTASIO CON ESPIRONOLACTONA, triamtereno o amiloride.
  • 84. LIDDLE, GITELMAN Y BARTTER • Que es la enfermedad de Liddle? • R = AD. El síndrome de Liddle (o SEUDOHIPERALDOSTERONISMO) es un trastorno AD caracterizado por HIPERTENSIÓN SENSIBLE A SAL con expansión de volumen, HIPOKALEMIA y ALCALOSIS METABOLICA, con ACTIVIDAD DE RENINA Y ALDOSTERONA EXTREMADAMENTE BAJAS y FG NORMAL. • Que es el síndrome de Gitelman? • R = Se caracteriza por HIPOKALEMIA e HIPOMAGNESEMIA, además de contar con HIPOCALCIURIA, perdida de sal y alcalosis metabólica con HIPERALDOSTERONISMO, HIPERRENINEMIA, asi como HIPERTROFIA E HIPERPLASIA DEL APARATO YUXTAGLOMERULAR. • Cual es el cuadro clínico del síndrome de Gitelman? • R = Debilidad muscular y DERMATITIS inespecífica. ESPASMOS DISTALES ocurren EN PERIODOS DE FIEBRE, vomito o diarrea. Algunos sufren condrocalcinosis. • Que es el síndrome de Bartter? • R = AR. Se caracteriza por HIPOKALEMIA GRAVE, perdida de sal y alcalosis metabólica con HIPERALDOSTERONISMO, HIPERRENINEMIA, así como HIPERTROFIA E HIPERPLASIA DEL APARATO YUXTAGLOMERULAR. Casi TODOS TIENEN NEFROCALCINOSIS por la hipercalciuria. • Cual es el cuadro clínico del síndrome de Bartter? • R = Tipo I y II: Las manifestaciones tempranas incluyen POLIHIDRAMNIOS, retardo en el crecimiento, polidipsia, deshidratación, avidez por sal y debilidad muscular. En el tipo III: Se manifiesta en la INFANCIA similar al Sx de Gitelman. El tipo IV: TODO LO ANTERIOR MAS SORDERA SENSORINEURAL E IRC TEMPRANA.
  • 85. SÍNDROME DE FANCONI • 114. Que es el síndrome de Fanconi? • R = Se debe a la DISFUNCIÓN GLOBAL DEL TÚBULO PROXIMAL. • 115. Cuantos tipos de Sx de Fanconi hay y que los caracteriza? 1) HEREDITARIO: Cistinosis, galactosemia, tirosinemia. 2) ADQUIRIDO: INTOXICACIÓN POR PLOMO O CADMIO, TETRACICLINAS CADUCADAS, tolueno (inhalar pegamento), agentes quimioterapéuticos como el cisplatino. 3) FANCONI LIKE: Pacientes adultos con disproteinemias como amiloidosis, en enfermedad de cadenas ligeras y mieloma múltiple. • 116. Cual es la anormalidad metabólica mas frecuente en el síndrome de Fanconi? • R = ACIDOSIS METABOLICA HIPERCLOREMICA.
  • 86. SEUDOHIPERALDOSTERONISMO TIPO 1 • 117. Que es el seudohiperaldosteronismo tipo 1? • R = AD/AR. También llamado síndrome de CHEEK PERRY y se caracteriza POR PERDIDA DE SAL, DESHIDRATACIÓN, presencia de hiponatremia, HIPERKALEMIA y acidosis metabólica con HIPERALDOSTERONISMO. • 118. Cual es el manejo del seudohiperparaldosteronismo tipo 1? • R = Dieta alta en SODIO.
  • 87.
  • 88. ATR • 119. Cuales son las tubulopatias asociadas a alcalosis metabólica? • R = Síndrome de CHEEK-PERRY(GITLEMAN), LIDDLE Y BARTTER • 120. Que es la acidosis tubular renal o ATR? • R = Grupo de entidades patológicas caracterizadas por defectos de transporte, ya sea REABSORCIÓN DE BICARBONATO (HCO3), EXCRECIÓN DE HIDROGENIONES O AMBAS. • 121. Donde se lleva a cabo la reabsorción de HCO3 y la excreción de H+? • R = HCO3 EN EL TCP Y H+ EN TCD. • 122. Que caracteriza a la ATR tipo I DISTAL O CLÁSICA? • R = ACIDOSIS METABÓLICA CON BRECHA ANIONICA NORMAL O HIPERCLOREMICA, HIPOKALEMIA E IMPOSIBILIDAD DE ACIDIFICAR LA ORINA A < 5.5 bajo el estimulo de acidosis metabólica intensa. • 123. Cuales son las bases fisiopatológicas de la ATR I? • R = 1) Defecto secretor (DEFICIENCIA EN LA SECRECIÓN DE HIDROGENIONES), formas dominantes, recesivas y asociadas a síndrome de Sjogren, LES, AR. 2) defecto de gradiente, en el cual hay un reflujo de los hidrogeniones normalmente secretados a nivel distal (anfotericina B) y 3) Incapacidad de generar o mantener una diferencia luminal negativa transepitelial distal como en la uropatia obstructiva, enfermedad de células falciformes, hiperplasia adrenal perdedora de sal. • 124. Como se diferencia la ATR tipo I de la tipo II? • R = Que estos pacientes a menudo PRESENTARAN NEFROLITIASIS Y NEFROCALCINOSIS.
  • 89. • 125. Como se maneja la ATR tipo I? • R = ADMINISTRACIÓN DE ÁLCALI, SE PREFIERE CITRATO DE POTASIO • 126. Que caracteriza a la ATR TIPO II O PROXIMAL? • R = Puede ocurrir sola o acompañada de otros defectos tubulares como síndrome de Fanconi y se caracteriza por una DISMINUCIÓN EN EL UMBRAL RENAL PARA EL HCO3, el cual usualmente es de 22 mml/L en infantes y 26 mmol/L en adultos. • 127. Que caracteriza a la ATR tipo IV hiperkalemica? • R = Usualmente ocurre con la presencia de IR MODERADA, sin embargo la magnitud de la HIPERKALEMIA y de la acidosis es desproporcionadamente extensa para el nivel de IR. • 128. Cual es la fisiopatología de la ATR tipo IV o hiperkalemica? • R = Defecto en la AMONIOGENESIS • 129. Cual es la principal enfermedad relacionada con la ATR tipo IV o hiperkalemia? • R = DM, además del uso de AINES, IECAS y ciclosporina.
  • 90.
  • 91.
  • 92.
  • 93.
  • 94. INFECCION RENAL • 132. Cual es el cuadro clínico de la pielonefritis? • R = Aparición de FIEBRE > 38.5, escalofríos, DOLOR LUMBAR, disuria, polaquiuria, HEMATURIA, NAUSEA, vomito, cefalea y diarrea. • 133. Cual es el manejo de la pielonefritis aguda? • R = AMPICILINA 1 gr + GENTAMICINA 1 gr por 21 DÍAS o TMP/SMZ 160/800 • 134. Cual es el manejo de la pielonefritis crónica? • R = Lo mismo que para la aguda pero con duración de 3-6m • 135. Cual es el manejo del absceso renal por S. Aureus? • R = NAFCILINA 2 g IV c/4 hrs durante 2-6 SEMANAS o CEFAZOLINA 1 gr IV c/6 hrs x 2-6 SEMANAS. • 136. Cual es el manejo del absceso renal por E. Coli? • R = AMPICILINA O CIPROFLOX CON GENTAMICINA
  • 95.
  • 96. LESIONES VASCULARES RENALES • 137. Cual es la causa de trombosis renal en niños y adultos? • R = En niños puede ser aguda debido a DESHIDRATACIÓN por diarrea y/o vomito. En adultos se relaciona con estados de HIPERCOAGUBILIDAD, sobre todo asociado a presencia de SÍNDROME NEFRÓTICO. • 138. Cual es el cuadro clínico del tromboembolismo renal agudo? • R = En caso de que la falla renal se acompañe de FIEBRE, dolor abdominal o en el flanco, NEFROMEGALIA, leucocitosis, HEMATURIA y aparición o empeoramiento de una PROTEINURIA previa y la VENOGRAFIA es el estudio de elección por que el Doppler tiene baja especificidad. • 139. En que casos se sospecha de estenosis de la arteria renal? • R = Se sospecha en pacientes con valvulopatias, IAM, arritmias o endocarditis • 140. Cual es la presentación clínica de la estenosis de la arteria renal? • R = Dolor en el flanco, FIEBRE, nausea, vomito, IRA. Se ha descrito EOSINOFILURIA en el cuadro renal agudo. • 141. Cual es la causa mas frecuente de nefropatía isquémica? • R = La estenosis ATEROMATOSA. Se consideran FR la edad, tabaquismo y ateroesclerosis difusa. • 142. Cual es el cuadro clínico de una nefropatía isquémica? • R = HIPERTENSIÓN INTENSA o de difícil control que pueden DESARROLLAR IRA AL COMIENZO CON IECAS. • 143. Cuales son los hallazgos de laboratorio e imagen en la nefropatía isquémica? • R = En los estudios de imagen hay ATROFIA RENAL y en laboratorio PROTEINURIA EN RANGO SUBNEFROTICO, con actividad de la RENINA ELEVADA. Siendo el estándar de oro la ARTERIOGRAFÍA.
  • 97.
  • 98.
  • 99. CASO CLINICO A 56-year-old man is involved in a severe motor vehicle accident. He develops ARF after admission to hospital. One of the possibilities for his ARF is posttraumatic renal vein thrombosis. Which of the following findings is most likely to suggest renal vein thrombosis? (A) white cell casts on urinalysis (B) heme-granular casts (C) heavy proteinuria (D) urine supernatant pink and tests positive for heme (E) specific gravity >1.020
  • 100. (C) Renal vein thrombosis is associated with heavy proteinuria and hematuria. Flank pain and pulmonary embolism can also occur. (Kasper, p. 1707)
  • 101. SINDROME UREMICO HEMOLITICO •En el SHE es común observar la prolongación del tiempo de? •Sangría
  • 102.
  • 103. NEFROANGIOESCLEROSIS • 144. Que es la nefroangioesclerosis? • R = Se refiere a la sustitución del parénquima renal por tejido fibrótico dependiente de colágeno, COMÚNMENTE VISTO EN LA HTA y que afecta de manera predominante a la vasculatura glomerular. • 145. Cuales son los hallazgos a la biopsia renal relacionado con nefroesclerosis? • R = Daño vascular con hipertrofia de la muscular y engrosamiento de la intima con posterior deposito de material hialino, predominando en las arteriolas eferentes. Glomeruloesclerosis con la consiguiente perdida de masa renal. • 146. Como se manifiesta la nefroesclerosis maligna? • R = TA >180/120 con daño importante a nivel de fondo de ojo y daño renal o encefalopatía como consecuencia de la HTA. • 147. Cual es el hallazgo en la biopsia renal relacionado a nefroesclerosis maligna? • R = Necrosis fibrinoide de las arteriolas aferentes y endarteritis proliferativa de las arterias interlobulillares con disminución de su luz e hipertrofia de las células miointimales dando apariencia en CAPAS DE CEBOLLA.
  • 104. HAS SECUNDARIA • 148. Cuales son las manifestaciones clínicas del feocromocitoma? • R = Triada de CEFALEA, RUBOR y PALPITACIONES aunada a HTA. • 149. Cual es el cuadro clínico del hiperaldosteronismo? • R = HTA, debilidad muscular, parestesias, cefalea, poliuria, polidipsia y edema de miembros inferiores. • 150. Cuales son los hallazgos de laboratorio en hiperaldosteronismo? • R = HIPOKALEMIA, alcalosis metabolica, HIPERNATREMIA e hipomagnesemia.
  • 105. CANCER RENAL • 151. Cuales son los factores de riesgo para desarrollar cáncer renal? • R = Tabaquismo, obesidad y abuso de AINES. • 152. Cual es la estirpe etiológica mas común en el cáncer renal? • R = CA DE CÉLULAS CLARAS asociado a deleción del brazo corto del CROMOSOMA 3. • 153. Cual es el cuadro clínico del cáncer renal? • R = HEMATURIA, DOLOR EN EL FLANCO Y MASA PALPABLE. • 154. Cual es la estatificación y pronostico del cáncer renal? • ESTADIO I: Tumoración de < 7 CM limitado a riñón con sobrevida del 95% • ESTADIO II: Tumoración de > 7 CM limitado a riñón con sobrevida del 88 % • ESTADIO III: Tumor DENTRO DE LA FASCIA DE GEROTA pero que involucra vasos mayores, glándula suprarrenal o una sección linfática. Sobrevida a 5ª de 59 % • ESTADIO IV: Tumor renal FUERA DE LA FASCIA DE GEROTA con sobrevida de 20 % • 155. Cual es el manejo del cáncer renal? 1. NEFRECTOMÍA EN BLOQUE ES DE ELECCIÓN 2. Ablación por radiofrecuencia en tumores menores a 3 cm 3. Interferon alfa que aumenta sobrevida a 54 meses
  • 106. CASO CLINICO A 69-year-old woman presents with left flank pain and hematuria. Physical examination suggests a left-sided abdominal mass. Computerized tomography (CT) scan of the abdomen reveals a 5-cm mass in the left kidney. Which of the following laboratory abnormalities might also be present? (A) polycythemia (B) thrombocytopenia (C) hypocalcemia (D) leukocytosis (E) high renin hypertension
  • 107. (A) This patient likely has hypernephroma (renal cell carcinoma). Polycythemia is caused by the production of erythropoietin-like factors. There is no relationship to hypertension. The tumor frequently presents as metastatic disease. (Kasper, pp. 541–542)
  • 108. CASO CLINICO A 64-year-old man is admitted for hematuria after slipping on an icy pavement. His physical examination is normal. A selective angiogram of the left kidney is shown. Which of the following is the most likely diagnosis? (A) renal cell carcinoma (B) kidney contusion and laceration (C) transitional cell carcinoma
  • 109. (A) The diagnosis is renal cell carcinoma. There is marked hypervascularity of the left kidney. The arteries are irregular and tortuous, following a random distribution. There are small vessels within the renal vein that indicate the blood supply of the neoplastic thrombosis involving the renal vein. The kidney is enlarged and abnormally bulbous in the lower pole. CT scans have dramatically decreased the need for arteriography in evaluating renal lesions. (Kasper, pp. 541–542)
  • 110. NEFROPATÍA ASOCIADA A PARASITOS • 156. Cuales son los agentes mas implicados en la nefropatía asociada a parásitos? • R = PLASMODIUM Falciparum y Plasmodium Malariae • 157. Cual es el cuadro clínico de la nefropatía asociada a parásitos? • R = ALTERACIONES HIDROELECTROLITICAS HASTA IR (Creatinina > 2) relacionado a formas maduras del parasito. • 158. Cual es el tratamiento de la nefropatía asociada a parásitos? • R = QUININA vía parenteral cuando hay resistencia a DOXICICLINA O TETRACICLINA • 159. Que ocasiona el P. Malariae a nivel renal? • R = Es de curso crónico y ocasiona nefropatía por paludismo • 160. Cual es el contenido de las células de estruvita? • R = MG, AMONIO Y FOSFATO
  • 111.
  • 112. ENFERMEDAD RENAL POLIQUISTICA • 130. Cuales son los criterios diagnósticos sonográficos para la enfermedad renal poliquistica AD? 1) < 30ª con al menos 2 quistes, uni o bilaterales 2) 30 a 60ª con al menos 2 quistes en cada riñón 3) >60ª con al menos 4 quistes en cada riñón • 131. Que cromosomas se ven implicados en la enfermedad poliquistica renal? • R = 4en ADULTOS y 16en FETOS • -A 30-year-old man presents with hematuria. His examination is normal except for an elevated blood pressure of 164/94 mm Hg. An ultrasound of the kidneys reveals multiple renal cysts in both kidneys. His father had a similar condition. Which of the following is not associated with this syndrome? • (A) liver cysts • (B) intracranial aneurysms • (C) autosomal dominant inheritance • (D) rheumatoid arthritis (RA) • (E) progression to end-stage renal failure • -RA is not associated with polycystic kidney disease. It has an autosomal dominant inheritance and about 50% develop renal failure by age 60 years. Cysts are also seen in the liver (more common) and pancreas. Also 5–10% of asymptomatic patients can have cerebral aneurysms. Renal transplantation is utilized in end-stage renal failure (ESRF). The transplanted kidney cannot be affected by the disease.
  • 113.
  • 114. CASO CLINICO Ten days after a kidney transplant, a 32-yearold man develops allograft enlargement, fever, oliguria, and hypertension. Which of the following is the most likely diagnosis? (A) steroid hyperglycemia (B) erythrocytosis (C) hyperacute rejection (D) acute rejection (E) renal artery stenosis
  • 115. (D) Renal scans initially show a reduction in excretion with cortical retention. This is the most common type of rejection. Most acute rejections will respond to immunosuppressive agents if diagnosed early. In contrast, immediate nonfunction of a graft can be caused by damage to the kidney during procurement and storage. Such problems are becoming less frequent. Obstruction, vascular compression, and ureteral compression are other causes of primary nonfunction of a renal graft. (Kasper, p. 1672)
  • 116. MISCELANEAS • El hipoaldosteronismo hiporreninemico que anomalía electrolítica da? • R = Hiperkalemia • El sx de Good Pasture , a que sitio se dirigen comúnmente sus autoanticuerpos? • R = Dañan riñón o pulmón. Los anticuerpos se dirigen contra la COLAGENA IV en membrana basal glomerular • Fisiológicamente que sucede con la disminución de excreción de fosforo? • R = Aumentan los niveles de Ca+ y se aumenta la PTH liberando el fosforo y por lo tanto hay niveles aumentados de PTH con aumento en recambio de hueso y lesiones subperiosticas. • Cual es el FG normal? • R = 145 ml/min • Cuales son las manifestaciones bioquímicas que tienen en común las ATR? • R = Acidosis metabólica hipercloremica hipopotasemica. • Cual es la triada clásica de la glomerulonefritis? • R = Hipertensión, edema y hematuria. • Criterios para dialisis? • R = ACIDOSIS SEVERA, HIPERKALEMIA, HIPERVOLEMIA, PERICARDITIS Y ENCEFALOPATIA • Cuando se considera oliguria? • R = < 400 ml
  • 117. • Que caracteriza a la cistitis instersticial? • R = Dolor al llenado vesical con mejoría al vaciamiento bajo tratamiento con Nifedipino y AINES. • Que cantidad de espermatozoides da oligoespermia? • R = <20,000,000 • Que enzima inhibe la finasteride? • R = Alfa 5 reductasa • Medicamento que utilizarías en la HPB? • R = Alfabloqueadores (prazocin) y RTUP • Cual es el valor normal de PSA? • R = 0-4 ng normal >4 inflamatorio • Cual es la estirpe etiológica mas común del cáncer vesical? • R = De CÉLULAS TRANSICIONALES
  • 118. • Como realizas el diagnostico de origen de hiponatremia? • R = ES en sangre, NA urinario < 20 mEq es debido a perdidas EXTRARENALES y Na urinario > 20 mEq son de causa renal • Como se encuentra el FENA en la IRA? • R = Prerenal <1% y postrenal > 2%. • Como son los cálculos de la relación BUN/ Creatinina con respecto a la IRA? • R = The ratio of BUN/creatinine is usually <10–15 IN INTRINSIC RENAL disease and >20 IN PRERENAL AZOTEMIA. • Qué medida de laboratorio es útil para el diagnostico de hipokalemia de origen renal o extrarenal? • R = El gradiente transtubular de potasio (TTKG), el cual si es < 2 traduce hipokalemia no renal y uno > 10 es debido a perdidas renales. EL K urinario <25 mEq/dia es no renal y K urinario > 30 mEq/dia traduce perdida de origen renal. • Que caracteriza al síndrome de secreción inapropiada de ADH? • R = HIPONATREMIA, osmolaridad <280, osmolaridad urinaria >150, función tiroidea y suprarrenal normal, Na urinario > 20. • Cual es el cambio por lesión mas común de la nefropatía diabética? • R = GLOMERULOESCLEROSIS DIFUSA, pero la glomeruloesclerosis nodular es característica • -A 63-year-old woman presents for routine evaluation. She has had diabetes for the past 12 years with complications of neuropathy and retinopathy. You decide to screen her for renal complications of diabetes. Which of the following findings is not compatible with diabetic nephropathy? • (A) nephrotic range proteinuria • (B) microalbuminuria • (C) hypertension • (D) red blood cell (RBC) casts in urine • (E) renal tubular acidosis (RTA) type IV • -Red cell casts are not seen in diabetic nephropathy and suggest another acute GN process. Nephrotic range proteinuria, type IV RTA (hyporeninemic hypoaldosteronism), hypertension, and microalbuminuria are all complications of diabetic kidney disease.
  • 119. • La acidosis tubulorenal a que anomalías se encuentra asociada? • R = Resistencia a la actividad de renina y aldosterona • -A 44-year-old woman is having symptoms of muscle weakness and fatigue. On examination, her blood pressure is 120/80 mm Hg, pulse 80/min, JVP 4 cm, heart sounds normal, and lungs clear. Her serum potassium level is 2.5 mEq/L, bicarbonate 18 mEq/L, and anion gap is normal. The urine potassium is 40 mEq/L: • -RTA types I and II cause hypokalemia with high potassium excretion (>25 mmol/L) and a low bicarbonate in the absence of hypertension. Diabetic ketoacidosis can also result in this constellation of findings. • • -A 78-year-old man is brought to the hospital because of nausea and vomiting. On examination he appears dry, his abdomen is soft, and the JVP is not visible. His laboratory tests reveal hypernatremia and his calculated free water deficit is approximately 3 L. In what part of the normal kidney is most of the water reabsorbed from? • (A collecting ducts • (B) proximal tubule • (C) distal tubule • (D) ascending loop of Henle • (E) descending loop of Henle
  • 120. • Cual es el manejo de elección en sx de Good Pasteure? • R = Plasmaferesis e imunosupresores. • Cuales son las causas mas comunes de IRC? • DM, HAS, Glomerulonefritis • CC sx uremico? • PRURITO que no cede con antihistaminicos (solo con dialisis), ESCARCHA UREMICA, color odre, ENCEFALOPATIA, asterixis, PERICARDITIS UREMICA, anorexia, nausea y vomito. ANEMIA, TROMBOCITOPENIA. • EN que enfermedad se presenta mas comunmente el hiperparatiroidismo secundario? • R = IR • Cuales son las manifestaciones clínicas del feocromocitoma? • R = Triada de cefalea, rubor y palpitaciones aunada a HTA. • Cual es el cuadro clínico del hiperaldosteronismo? • R = HTA, debilidad muscular, parestesias, cefalea, poliuria, polidipsia y edema de miembros inferiores.
  • 121. • Cuales son las 3 manifestaciones renales asociadas a endocarditis? 1) Secundaria a inmunocomplejos presentándose como Glomerulonefritis membranoproliferativa 2) Secundaria a medicamentos que pueden ocasionar nefritis instersticial aguda o NTA 3) Secundaria a embolos sépticos • Clinica a nivel renal de la endocarditis? • R = Hematuria, cilindros eritrocitarios, HTA e IR. • Que medicamento se utiliza en hipercalciuria por aumento de la absorción del calcio intestinal (> 200 mg/24 hrs)? • R = Fosfato de celulosa por que se fija al calcio e impide su absorción • Que tamaño deben tener los cálculos urinarios para poder pasar sin causas obstruccion? • R = < 6 mm
  • 122. • Cual es el tratamiento de primera línea de cistitis intersticial? • R = No hay cura. Amitriptilina y nifedipino • El 90-95% de los tumores malignos testiculares primarios son de células germinales “seminoma o no seminoma” , a que estirpe pertenece el resto? • R = No germinales, leydig, sertoly y gonadoblastoma. • -An 18-year-old man is found to have metabolic alkalosis and hypokalemia. This could be secondary to: • -Bartter syndrome and Liddle syndrome can be inherited in an autosomal dominant fashion. Patients with Liddle syndrome have hypertension, whereas those with Bartter syndrome do not. In both syndromes, hypokalemia is prominent • -A 28-year-old man presents with a kidney stone. He is married to his first cousin, and 6 months earlier, his 8-year-old son had a kidney stone as well. The most likely diagnosis is: • -Many of the listed disorders can cause nephrolithiasis, but cystinuria is the most common cause of stones in childhood. It is a common inborn error of amino acid transport and is inherited as an autosomal recessive trait. The disorder affects transport of all dibasic amino acids (lysine, arginine, ornithine, and cystine) in the kidney and the gut, but symptoms arise from the overexcretion of cystine because it is the least soluble.
  • 123. CASO CLINICO A 15-year-old boy develops renal colic. The stone is not recovered, but urinalysis reveals hexagonal crystals, and a cyanide-nitroprusside test on the urine is positive. Which of the following is the most likely diagnosis? (A) cystinuria (B) thalassemia (C) hereditary glycinuria (D) primary hyperoxaluria (E) sarcoidosis
  • 124. (A) Cystinuria is a congenital disorder associated with decreased tubular resorption of cystine, arginine, ornithine, and lysine. Only cystine is insoluble and is the cause of renal calculi. The typical hexagonal crystals are most likely to be seen on an acidic early-morning urine specimen. Apositive cyanide-nitroprusside screening test should be confirmed by chromatography. (Kasper, p. 1714)
  • 125.
  • 126. GLOMERULOESCLEROSIS FOCAL • En que lesiones renales se observa glomeruloesclerosis focal? 1) Adicción a la heroína 2) Rechazo de injerto renal 3) Reflujo vesicoureteral 4) Agenesia renal unilateral
  • 127. SONDA VESICAL PERMANENTE • Cuales son las medidas a seguir en un paciente que presenta sonda vesical permanente? 1) Administrar ATB cuando manifiestan clínicamente una infección 2) La bacteriuria suele desaparecer al retirar la sonda 3) Los ATB con fines profilácticos no evitan la bacteriuria 4) Los antibióticos están indicados cuando se procede a cambiar de sonda 5) Si el paciente orina espontáneamente es preferible usar un colector
  • 128. AMINOGLUCOSIDOS • Que factores agravan la nefrotoxicidad por aminoglucodidos? 1) Edad avanzada 2) Tratamiento prolongado 3) Diuréticos 4) Insuficiencia renal
  • 129. YODOHIPURATO 131 • La radiografia radioisotopica marcada con yodohipurato 131 de que patología tiene interés diagnostico? • Hipertensión vasculorenal
  • 130.
  • 131.
  • 132.
  • 133. Consejos 1. Leer diariamente y en bloques 2. Adiós Partys un tiempo 3. Has ejercicio y come bien durante el estudio 4. Toma algún curso bueno si tienes la posibilidad 5. Ten Fe.
  • 134. BIBLIOGRAFIA • EXARMED • PAPADAKIS • CTO • HARRISON • AMIR • USMLE STEPS

Notas del editor

  1. PPT PURPURA TROMBOCITOPENICA TROMBOTICA